Anda di halaman 1dari 106
Contents |. Simple Beams II: Energy Principles |. ‘Two-dimensional Problems ‘Contents 44.1 Closed profiles Si General, Statics 32. Large Curvature. 53. Small Curvature : ‘Sa Deflections of Curved Beams 3:5. Exercises to Chapter S . 6.1 Reciprocity Theorems of Be 62. Theorems of Engesser an 6.3. Statically Indeterminate 64 The Complementary 65 Strain Enerey of Beams 6.6 Application to Beams .. 6.7. Exercises to Chapter 6... 7.1 Plane Stress and Plane Strain .. 7.2 Body Forces Derived From a Pot 73. Plane Problems in Polar Coordinates 74 Exercises to Chapter 7 “sR ‘ oe Sa ae 149 General Remarks i 149 92 Bifurcation Problems with Fnite Degrees of Freedom we 150 93. ASnap-Through Problem. .. 152 94 Column Buckling 153 95 Exercises to Chapter 158 Contents XI 10.3 ‘The Euler-Lagrange Equations .... 10.4 The Lagrangian Multiplier Method... 11, Oscillators With One ieee eee ‘ tion 12, Systems of Several Degrees of Freedom 7 12.1 A Typical Example " 7 12.2 General Equations and Solution. 178 12.3 Forced Vibration with Harmonic Ex 179 12.4 Exercises to Chapter 12 183 13, Answers to the Exercises ....... ae 185 1. Basic Concepts of Continuum Mechanics 1.1 General Remarks In Chapter 1, we present general concepts and definitions that are fundamental to ‘many of the topics discussed in this course. Mechanics has been defined as the study of forces and motions. define motions as the change in postion ofa body, in time, and with res frame of reference. The total force F on a body B is the vector sum of ll the forces exerted on it by different a 1.2 Stresses AAis called the stress or s(0)= Jim, 3a aa cy inthe element, where te orientation of area Ais designated by the unit vetorn. tonand perpendicular to n. These are the projected stresses, namely, normal stress 2 nn nm = o(n)n = (s-n)n, a2) 2 1. Basic Concepts of Continuum Mechanics Fig. 1 Section surface and related stress vectors and shear stress ony one =8—o(n)n =x (sn), with og m1 = 0, Obviously, on, can be once more resolved into two components {in two arbitrary directions inthe area element ‘When different sections are passed through the same point characterized by vec- torr, different stresses are found in areas dA of different orientation. Especially, in 8 aytem of etngulacresian corns, when n= 6, Se = Or202 + Oxy y +020: = ar Ont. a4 Using Einstein’s summation convention, Eq form 4) may be written in a much shorter Be = ose) 1= 3,2 In an analogous manner, we may consider two additional sections perpendicular to the y and axes y= eH0% 8: = 0n0. It can be shown that in an Eulerian space the state of stress at some uniquely determined by these three stress vectors from three orthogonal sect their 9 components, which are the 9 components a, i,k = ,y, of the stress tensor c. The four sections described above constitute a small tetrahedron at point + with three faces lying in planes perpendicular to the «, y and 2 axes, and a fourth ‘one with area da characterized by unit vector n. Thus the three vector equations (1.5) and (1.6) may be summarized to give s(n) = sin, ay and further s=o', o= oun, where now the second-rank stress tensor ois defined as a linear mapping, and = one Ser Kk=2,y,2, a9) 12 Stresses 3 with the symbol @ designating the tensor product (dyadic product) of two vectors. ‘tis sometimes usual to write this in the form of a matrix :) ; (1.10) ibscript indicates the section in which the stress acts, and the tion of the stres Of the stress tensor =o" + on =n follows from the eq Of the moments of the forces acting on a volume 11) reduces the number of independent components the more specific case of plane or biaxial stresses, where the the = direction are assumed to vanish =0. a2 For this plane stress case, we will now discuss the influence of the direction of the ‘Transformation ofthe stresses Let y be the angle of rotation, and dA be the area of the inclined face, Then, the Gn = Orn 008 + oyy sin? p+ 2ory sin pcosy 04 = (Ory ~ oz) Sin 9008.9 + ony (cos? p — sin? y) a3) 9 = Ors 8in” p+ ayy C08? y — 2ozy sin yeosy. Equation (1.13) is derived from a prism containing a face y = const. These equa- tions may be written in a more compact form 4 J, Basic Concepts of Continuum Mechanics 4) From these transformation formulas, we can conclude the following relations 1, Arotation of ¢ = nz does not alter the components. 2. Arotation of = 1/2 causes ee = Oy, Ogg = Fx) 99 = —Cry- (ts) 3. Extreme values of ox» and ogg, and vanishing shear stresses ay are attained foran inclination of o= arctan 221. p ee Oy ‘The orthonormal base vectors (€1,€2) related with these directions form the ‘ineipal axes of the stress tensor with the extreme values 1 1 O12 = 5 (22 + Ow) * 5 (ee — Oy)’ +403,, ay (2.16) where 1 > ap. Since the solution of Eq, (1.16) is in general not unique, the following table may help in finding the magnitudes of the inclination angle Gaz 2 Oy | Fy 20] 0S RST Oxy <0) —7 detox — 78x) = 0 27 holds, where 1 is the unit tensor, and 1 for i= s 1.28) Sue (Genet (1.28) is called the Kronecker Delta. ‘This cubic equation (Eq. 1.27) 6 1. Basic Concepts of Continuum Mechanics 3 — So? — Sjo — $5 =0 (1.29) has three roots, which are the values of o for which Eq, (1.24) holds. Such roots, ‘are known in general as the eigenvalues of the matrix or, and in this particular case (1.30) (31) (1.32) Il = 3 (01 ~ 05) (1.33) when we define 01 > 02 > 03 For a plane stress state, we find $$ = 0 (since oy = ‘equation (1.29) turns y) and thus the cubic (1.34) T=O-Oml, Te =i —Ombn (1.35) wwe can introduce a stress deviator 7, such that with the mean stress or hydrostatic stress 1 om = 351+ 136) the stress tensor can be split into two parts Its obvious that T= =0 Ty = ram = S249 a3 Ty = Tarastsi = Sa ~ $1S2-+ BSP are the invariants ofthe stress deviator. ‘1.2 Stresses 7 22 = 16) MP2, yy =60 MPa, ses and planes, (b) the stresses on (6) the maximum shear stresses. which gives {G@) To locate the principal planes, we make use of tan 29 =0,8 + go = 19,33", 64,03; 01 = 174,03; 02 = 45,97. [Mpa] bstituting 2 = 90° into these equations gives a2 =150, oyp=70, og =—50. [Mpa] 2) The angle ofthe plane of maximum shear stress is found to be p= 19,537 + 45° = 64,35°, rae = 64,03. [Mpa Farther, we find exe = opalpmpn = 110. Mpa} sxample 1.2: 1a problem of plane stress, the normal a stresses 07, 077 and 01 have been mea- different directions. Principal stresses and planes, and the maximum shear stresses, with a; =100MPa, 11 = 50MPa, e111 = 100 MPa. 2y has not been measured, we frst determine it from Eq. (1-12); through e ae ey ' O82 = 5 (xe + Oy) + O0y ee: 2y ~ 50 Mpa With this information, we find from (1.15), and 8 1. Basic Concepts of Continuum Mechanics ‘Example 1.3: A plane sheet (plane stress state) is subject to loads as shown in the figure. Boundary AC is stress free. Determine: (a) the angle a, (b) the principal stresses and planes, with: (41) = 40 MPa, a) = 20 MPa Solution: From the equilibrium equations, we find am sina + 7 cose: = 4) 2, T= e%1aB%80) ec) cosa +Tsina = 0 tana = -o(80)/7 7 =20V2MPa, a = ~35,26°. ‘Then from Eq, (1.15) 12 = 30430, 01 =60, o2=0. [MPa] Example 14: ‘The following matrix of stress components 622 ay=(20 4) (MPa) 240 ‘describes the stress state at a point of a structure. Determine the principal stresses, and the maximum shear stress, Solution: From the determinant (1.27) G-0 22 Perce 2 4-0 we find the cubic equation = Go? +640, (+ 4)lo 8-2) =0 0= with solutions: o1=8, 02=2, o9=—4, [MPa] Ile = $ (01-03) = 6. [MPa] 13, Stenins 1.3 Strains Basing» roses of fonation a one ps of ts Dody under consideration ‘ways. For each axis ith coordinates (x,y, 3) undergoes a displacement 1 with components jon of a volume element dr dy dz may be deseribed in various its increase in length vided by its original length as its extensional or tensile strain Deformation ofa rectangle be described by the Figure 1.3 showsa rectangle ABC and the quadrilateral ABC" into which ‘thas been deformed during this process. In tis figure, the deformed element has ‘been moved back so that its lower left comer isin the position which it had before deformation. Consequently, the vector BB" has the components oe oa Fide and GE ae, ‘when the partial derivatives ofthe displacements are small compared to 1. The elon- "gation of the line element AB is aap AB! ~ AB =F dz, (1.38) and the strain ¢2» ofthis line element is this quantity divided by the original length AB= de fee = Gee or (1.39) ‘Similarly, the strains ¢,, and e,, of the line elements dy and dz are obtained Oy us ey = Oe, ta = Oy? ‘Oz (1.40) 101. Basic Concepts of Continuum Mechanics In two dimensions, there is only one further strin compor decrease tight angle betweea the sides AB and AD of thclemon, Wo define on f 1 (Atty toy= 9 (naan +1000) =5 (B+ Fe) aay Thedifeence esi ue 2 \ Oe Oy (1.42) defines the average rotation of the element. In three dimensions, there are two additional shear strains (1.43) (1.44) ees (14s) eel as 1.46) ip are the components of a second-rank tensor, the (infinit i se nor, the (infinitesimal) stai exe", cune eT, cu eu (47) is symmetric. Since @=-a", a =—ay, “a= a, aie =—axi, 4 =0, (1.48) ‘xis an antisymmetric devia ‘Sometimes itis more convenient to describe the strains in alternative coordi pie ener ania eee Moe eae 1. Cylindrical coordinates (r, 9, 2) Our ey 1 Ou Ou, f= Ge. toe = ty mee Ort eH ete ae eas oe (1.49) 1.4 Compatibility Conditions 11 (1.50) as) Sun 8a . «1s2) He 14} «is a symmetric second-rank tensor, and thus follows the same rules, and has the same properties as the stress tensor. This means that we simply take the relations and formulas which have been derived forthe stresses and replace herein the stresses by the strains (or any other symmetric second-rank tensor). Defining the strain deviator 1 1 qaen pel, =e gedit ‘we can also split up the strain tensor into a first part ¢ = €4 describing the volume dilatation, and a second deviator ix describing the volume preserving distortions. j 1.4 Compatibility Conditions stain to be derived from ony three this means tat the stains cannot all coordinates; they are connected by a corresponding number of equations. Physiealy, it means thatthe strains must be compatible; the deformed elements must together Th three dimensions here exist 6 compatibility equations Pen , Penn Pein at + Get —2Badm 89 hh a 121, Basic Concepts of Continuum Mechanics eee Gate | Or8z, — Oz2 — Broz, = LH (1.55) ‘where here the summation convention is excluded for the given indices i and k, and i, respectively. ‘The six equations cannot all be independent sufficient to determine the six strain compor is easly verified that among them three iden cif they were, they would be ‘of course, is impossible. It ld (Bianchi’s identities), ibject to a sum of forces F of two kinds; long led as a continuum occupying a region V. then felt throughout V, while the short-range forces act as contact forces on the boundary surface A. ‘We start from Newton's second law of motion D AP = dFy + dF 4 = 5 (dmv) (1.56) Which here is given fora volume element ddy d= of the body, and where dm = paV = pdzdydz st) is the mass of the clement, and p is the density (mass per unit volume). Thus, any volume element dV experiences a long-range force dy = dmb, sum of body forces. (158) Any oriented surface element n dA experiences a contact force dP, =s(n)dA. 59) Die) Ia = ¥ = tt denotes the acceleration field, where (*) = —5** denotes the material time derivative, then the global force equilibrium of motion (balance of linear momentum) is Jovavs [atmaa= [oaav, (1.60) : 4 é erin fe contention of mass (din) = 0 uke ino comin, Equation (1.60) is known as the first of Euler's equati (balance of angular momentum) has already been used ( ‘symmetry of the stress tensor. With Eq. (1.8) and using Gauss's divergence theorem fosmaa= [Beav, (161) A a 1.5 Equations of Motion 13 Eq, (1.60) becomes [ {FE +e -pa)} av =0 t (1.62) ‘This equation embodies a fundamental physical law and thus must be independent of how we define a given body and therefore must be valid for any + including very small regions. Consequently, the integrand must be zero, and thus we obtain the local force equation of motion Boj Fe + bs = pay In usual notation, we write Bees , Oye , Oz Oz * by * de describing the local balance of forces in x direction, and +pbs = par Bony | Boy , 06 sn 4 OS 4 + py = pty eae 4 ae 4 9 4 ob, = pa, do, do, do; ieiellton he a ue ees 4 Seas 4 Fe phy = iny and z directions. For vanishing acceleration tems, inown asthe equilibrium equations of continuum mechanics For cylindrical coordinates, we find 0m , 1 B6—r , Ooer , 1 en pA Daren Oar. Geto ebb 20 mearenvty * Os +5 (Om Ope) +P dare , 1 B06 , Oo , 2 rey 1d0ee , Bowe 2 oh, <0 et rag tas pote Pies yes, Wet. Dore ti =0. Bor, | 1 dog | 1 Door or + 5 Op 7 ring OO 1 " (1.63) (1.64) (1.65) (1,62) and (1.63) are (1.66) +5 (201 = O99 ~ 790 + 20r) + pbr=0 Boe , 1 dove 1 + 2 (cegeoty—so9cot e+ Bory) + phy =0 Op * rang 00 Bore 1 do90 , 1 O00 , 1 sat + = £ (Bore + 2ap0 by = 0. anita rampipea ti Crete tr) te (67) 141, Basic Concepts of Continuum Mechanics 1.6 Energy ‘We now consider our body in a state of motion. A volume clement dV of this body ina time interval dt experiences an infinitesimal additional displacement du= vat. (1.68) ‘To determine the energy balance, we again start with Newton's second law of motion (Eq, 1.56) and multiply this equation by du. The global balance thus becomes [ar au= fans au= [pv auav. (1.69) ‘ v é Using the divergence theorem, we find —in index notation ~ 0 [{ FE +om}onav = fomsuav 4.70) and further a a 1 ff Bag lie di) — rao ( due) + by ou} ave 5 /eDuiav. am v ‘The first term gives (again using the divergence theorem) a [lowsuyav = [ovdunaa =dAy 72) v a the increment of the external work of all short-range forces, Apparently, the third term describes the incremental work of the long-range forces [orauav = aay. 73) ‘ Duc to the symmetry of the stresses, the second term is, a au ger(din) AV = fou dewaV = a, am) v , v the increment of the intemal work or energy related with the deformation of the body. Thus for rigid bodies this term must vanish, Finally, the right-hand side of Eq, (1.69) gives the increment ofthe kinetic en- ery 1 3 /eDweav = ae. as) ¢ 5 Ths, we find GA= dda + dy = aW 4 db, (4.76) 17 Principle of Virwal Work 15, the energy balance of mechanics. Any change of the work dA done by the short- range and long-range forces causes a change of either the internal work d1V’ or the kinetic energy dE. For static problems, with d = 0, this means that any change of dA causes a ‘change of dW, or simply aw — A= aW— A) =0, expressing the stationarity of the function W’— A. This means that fora system in equilibrium the function W — A attains a stationary value, 1.7 Principle of Virtual Work ‘The concepts of virtual displacements and virtual work are usually introduced dur- 3 during a virtual displacement is called viral work Similar as before, we introduce viral displacements bu, - in index notation - ‘which are related to virtual strains dee by 1 Abu , Au Cake { Ox * Oz }. {As im the preceding Section, we start by multiplying the equilibrium equations by ‘uy and ths ative at (78) {Zt +em}sucav -0 79) y and further a a {{aqouon) ou (un) + nbniue} av =0 (180) ¥ ‘The first term gives (again using the divergence theorem) Per ie famctas as, a the virtual work of the external forces, prescribed on the surface of the body. Feo ieee fe te bee forces f prturavlsaiy (1.82) ‘ 16 1. Basie Concepts of Continuum Mechanics Due to the symmetry ofthe stresses, the second term is [ou geatouyav = [ouseieav =o, (1.83) v 4 v the virtual work of the internal forces. The principle of virtual work may thus be stated 0=~ fou denav + [rntupavs [oxsueaa v ‘ 4 =-0W +5Ay +5Aa, (1.84) similar to Eq. (1.75), and finally SW 5A = 5H — A) 0 (1.85) lation of mechanics problems than the later. 1.8 Exercises to Chapter 1 1 S= Si, 53 = 5(S2— Give the principal invariants T Sj = dete = qt = 35,52 + 28s). of the stress deviator 1 =o-oml, om= 5 a0 5a as functions of the principal invariants of the stress tensor o. Show that 7 and hhave the same principal directions and that the eigenvalues r; of 7 are related to the ‘eigenvalues 0; of o by the expression 1.8 Exercises to Chapter 117 = 0— Om. Use the substitution T TH 2/5 Ts cosy “tofind the roots of the characteristic equation P-Tjr-T=0 ‘of the stress deviator, and hence show that the eigenvalues of are : aint (si+2 53-4385 omy.) (i=1,2,3) 1 2785 + 9835, + 257 a Qn wo! ee ee ane 4) the principal stresses and planes, 'b)the stresses on an element rotated by an angle of 45°, “€) the maximum shear stresses. ‘Determine these results also using Mohe's circle. 18 1, Basie Concepts of Continuum Mechanics Problem 1.4: Compute forthe plane deformation field e744, yom Eee t¥e Is the sin easore, 2. the rotation tensor ay, and 3. the principal values and directions of . Problem 1.5: For the 60° strain rosette shown in the fig- ture, gage A measures the normal strain = and C measure the strains &y spectively. Compute the strain tensor ©i4, its principal invariants, and the principal strains € and e2. 2. Elastic Material (2.1 Hooke’s Law ‘The preceding discussion of stress and strain is concemed with kinematics and stat- ‘elation between action (stress) and reaction (strain) o=Ee Qn for the description of a one-dimensional problem. It may be easily verified that the most general law which corresponds to (i) and (it) is ‘Be, 201 —v(02 +05), Bey = 02 ~v(03 +01), (2.2) Bes =03-v(01 +02). lus) and Poisson's ratio v must be According to (ii) E and v are independent of time; according to (iv) they are inde- ofr. 202. Blastic Material Hooke’s law may be further transformed to an arbitrary orthogonal frame. The ‘most general linear relation is - in index notation ~ 23) -rank linear material tensor with 3 = 81 coordinates. Some where Cyne ‘symmetry properties reduce this number to 21 independent material parameters for ‘general anisotropic linear elastic materials (6 independent parameters, respectively, for plane stress state) ‘Equation (2.2) may uniquely be inverted to give 24 = Syn on, 4) ‘where the compliance tensor Susnt = (Cizat) 2s) is the inverse ofthe elasticity tensor. For an isotropic, and linear elastic material, the number of independent material parameters may be further reduced to two. We get on 24(«a ae etx) 2.6) and the inverse 1 ae en ‘where the shear modulus Gis given by E SER 28) ‘When stress and strain tensor are split into deviatoric and hydrostatic parts, Hooke’s law may be written in the particularly simple form om= Ke, T= 26 tie, 29) where E 3-2) is the bulk modulus, and e is the volume dilatation. We emphasize that with Eqs. (2.9) the volumetric (dilatational) and deviatoric (distortional) parts of the stress- K= 2.10) are measured experimentally 8 replaced by the Lamé co- gut = (Sint + Sudsu) + Adis b0 - ut) 22 Surain Energy, Complementary Energy 21 Introducing this. mn into Eq. (2.3), constitutive relation (2.6) may be replaced ipler form 4, o=mjetrel, 2.12) hold v= ut (2.13) 30" ee ‘The Lamé coefficients are easily deduced A= ae us =G. (2.14) Waar’ "= 24H Inthe sequel, we prefer to use the material parameters E/and v, and G, respectively, rather than the Lamé coefficients. 2.2 Strain Energy, Complementary Energy In Section 1.6, we introduced the increment of internal energy dW as aw = fawam= [ou deudm Qs) ‘The local quantity herein dw = ondcie 216, ? vill be called increment of the specific intemal energy, i. the increment of the intemal energy per unit mass. ‘We can now introduce into this relation the additive split of both tensors. and ‘dei, into their deviatoric and hydrostatic (volumetric) pats and obtain di = dy + ding = Fm de-+ 3 a dra 2.7) ‘where the first term dioy describes the energy (work) related with the volumetric pose any initial state as a “natural state”, i. that at time t = to: it, = 0 is assumed, then using the generalized Hooke’s law (2.6), integrated to give 1 a Ke? + Gyn (2.18) Temay easily be shown that this specific strain energy can also be written as plc) =G (sew +7 “sy 2) r 2.19) 222, Blastic Material ‘Using Eqs. (2.9) and (2.7), this may also be expressed as a function of the stresses pulou) = pe (ou) = 552 02+ Ge Tar 22 et pruleix) = pe" (on) = 3 (ou -*% a) 8 20 Since the sum of Eqs. (2.21) and (2.19) (or (2.20) and (2.18) gives eu) +0*(6u) = Fence, 222) the latter is called specific complementary energy. 4 paw" | et Fig. 2 ® Strain and complementary energies in uniaxial tension SEE 5 Introducing here in analogy with Eq, (2.16) dwt = eadow, pe that pw(eix) and pw* (9x) are complementary to each other in represent 2.3 Fundamental Equations {In addition to the field equations a set of boundary conditions is needed: cither the displacements u (or derivatives of its components) or the surface tractions 8 must bee prescribed on the surface ofthe body. ‘A reduction of the 15 field equations can be accomplished in various ways, two ‘of which will be mentioned here: (The first of the reduced systems is obtained, when we introduce the kinematic relations and the stress-strain relations into the equilibrium equations. ‘This yields three equations 24 Influence of Temperature 23 1 de by Aut = oe tg 7% (2.24) (2.25) (Gi) The second system of equations are the six Beltrami’s equations forthe stresses. 3 Pom Ara +55 Sate +0( at mt it) <0, 27 where i Om = 508 (2.28) is the mean stress (hydrostatic stress). for sotving the n we prefer is mainly related with the given boundary For problems with preseribed displacements the frst way would be advantageous, whereas for prescribed tractions the second way could be more favourable. 24 Influence of Temperature _ A uniform, ic. homogencous temperature field causes a volumetric change of the ‘body under consideration. For the different materials, we find Where a is the linear expansion coefficient of the material, and @ is the increase in temperature, ‘The inverse of Eq. (2.29) is v Ez oR 26 (ca +e bu) ~ pO hn, 230) _ and, consequently, Eq, (2.9)s changes to om = K(e~ 300), 3p 24 2. Elastic Material ‘whereas the deviatoric part (2.9)2 remains unchanged. For a rectangular cartesian coordinate system, we have (Eq. 2.29) ae ts Hen om )}+00 yy — (tas + O21 -)} +00 ew = é = {0 Mose +ow)} +09 e ‘ eH (2.32) and from the inverse relation (Eq. 2.30) iw ree {(1.=v)e00 +¥(y +222) ~ (1+ ¥)0} en pl 2 + a2) - (1+ v)a8} ose = AE f(t reen + Ucan + éyp)~ (14+ 0100} at waetOea oy =2GE yn Cu = 2G 2.5 Hooke’s Law in Two Dimensions (2.32) and (2.33) to two dimensions, letting plane strain). The equations relating stress fw = Fn — Ver) +08 234) 1 toy = 5g Orv and, further, fas =F Cae toy) +08 in the = direction. 2. Plane strain: ew = 3g {0-Mow Hee} + ie ey = 9G Ov -and in the 2 direction. © 22 = (dex + Oy) ~ Ea. is obtained by the procedure: E1427) v par) then drop the primes. SB => Example 2.1: A reinforced concrete plate has been built at a temperature of +5°C. What happens if the plate is heated to a con- stant temperature of 55°C? ne the stresses in the plate, and the horizontal forces in the supports if ‘expansion is restrained: : ia 2.110! MPa, a = a ty vem, 2.5 Hooke’s Law in'Two Dimensions 25 (2.35) = 5 {0 -Yeee ~ voy} +14 1)08 1 +v)08 2.36) 237) The problems of plane stress and plane strain, although different in their basic equations, are not independent of each other. When a plane stress problem has been solved, the solotion forthe corresponding plane strain problem may be derived from + asai(ity) 238) Conversely, if the plane strain problem has been solved the solution for plane ity we (2.39) asa Its easily seen that neither substitution changes the shear modulus G. ee : We introduce a cartesian coordinate system with the ‘and the y-axis in thickness direction. From Hooke’s horizontal direction, (E4234) pacer for Ore = Oe = == 08 =—-15MPa f= —1800 kN Example 2.2: ‘A quadratic piece of rubber with dimen- sion a is pressed into a quadratic hole. rigid, Determine the stresses in the rubber. ‘What happens if Poisson's ratio v tends 100.5? Solution: Due to the assumed rigidity, we find fax == 0, > Introducing o.. = ~F/a?, we determine from Eqs. (2.32) vr caer earth and furthermore (14 y)—2) F —v Ba ‘Thus, for v = 0.5 the material behaves like a rigid body. 2.6 Strength Criteria reached. This situation can be avoided with a judicious choice of a safety factor in 26 Suength Criteria 27 Jou), reaches cr. That is, ey =O1, Of dq =—93. (2.40) ‘These equations ignore the intermediate principal stress (¢2) and assume, along with all other criteria that the tensile and compressive yield stress are equal. teaches a critical value. In the simple ten have i Frlow = 52 (2.41) _ Equating the shear stresses for the uniaxial and multiaxial cases, leads to the Tresca criterion 1 = 0,02 = 03 = 0, we ug = 27h = 01 — 03 2.42) (iii) Total Strain Energy Theory (Beltrami, Haigh) ‘The theory assumes that yielding commences when the total internal energy stored attains the value of if internal energy for uniaxial yielding. Thus from (2.21) ge-as Ds 243) ete its ashe In the uniaxial ease (o1 = 0,02 = 3 =0), we find Pwo" (ix) = pieya kes puro) = 307, em thus, equating this expression with the energy for the multiaxial case, we find (2.45) expressed in terms of principal stresses. (Gv) Shear Strain Energy Theory (Maxwell, Huber, v. Mises, Hencky) ‘This theory assumes that only the distortional part of the internal energy will con- tribute to yielding, and thus from the second term of (2.20), we find pw (ix) = Gq Te (2.46) ‘the second invariant of the stress deviator. From the uniaxial pw'() = G57 ar ‘Again, equating this expression withthe distortional part ofthe intemal energy for ‘multiaxial problems yields Example 23: ‘A closed thin-walled tube (container) with: radius R, thickness H is subjected to internal pressure p, and additional torsion with shear stress 7 (a plane stress state is supposed). The stresses in a cylindrical coordinate system are then 26 Strength Criteria 29 cpp =0,=p%, on =0,=h05, ope = oe = Oe = PE, Ou =Or= 505, Op =T- Determine the equivalent stresses according to the different criteria: Maximum principal stress, (b) maximum shear stress, (c) total strain energy, and > Xa Fip23 Principal stresses vs, shear stress © m= 5 (oetas)t 5 Vl, — 0.) 47 ae ‘in normalized expressions aia=34VIFF, (@)=Ale)/0, different equivalent stresses are oq so 3 4g 234 ITF ae _ _ [34viFF, o seyle2 +472 <1, Beay RET St, hog gV He TIES, vi v3 @ F344? Solution: ‘We first divide the cross section into two rectangles of dimension a « 2a, and deter- ‘mine the coordinates of the common centroid with the help of relations rid: ' = 0/2202 + 20-208 5 paar a? Te Dic Ai 4 i = a da? a = +20? + 30/220 _ 5 ie Where the y/and 2 coordinates, respectively, are the distances measured with respect {0.a parallel system with its origin in the upper right corner of the cross section. Now ‘using the parallel-axis theorem (Eq. 3.24), we can determine the moments of inertia, ‘with respect to the 1 and = axes, respectively, as the sum of the moments ofthe two, separate rectangles 423. The Theory of Simple Beams I Due to the symmetry of the two rectangles their product of inertia vanishes and we thus get from the parallel-axis theorem (Eq. 3.243) t=0-20(I) (Fe) 0-204 and finally, since J > Jay SE ont ah Aa qZal, b= Gat. Likewise, we compute the directions of the principal axes by applying Eq. (1.16) to the moments of inertia, : . 1 . “oc < re Juv Jez and y<0 > -F Coy = Ons oF ‘Thus the resultant shear stress 7 is determined from 7 r= oh, +08, = omy + (2) and the direction of runs with B tH parallel to this boundary. tana = St = Example 3.4: For a circular cross section with radius Rand area A= R?, we find mt} ee See pe 8 A “with the maximum value of the resultant shear stress 49. a ie 48 3. The Theory of Simple Beams I fon the y axis (at 2 = 0). Finally, we will consider a particular class of beams, known as beams of thin- (the cross section has a thickness 6 which is small compared to its overall height or width, and Gi) the cross section is either open (a) or closed (b) mq [ameape Ta) He i} @ | 2 | erat | oy ROL” adit pL | » » Fig. 3.7. Thin-walled cross sections: (a) open, (b) closed ‘Beams of thin-walled cross sections are widely used in engineering application. 4 mechanical description of the profile of the cross section. These are the ¢ axis, following the (dashed) cross-sectional centerline, andthe 7 axis perpendicular to ¢. ‘This new (rectangular) description allows us to modify the assumption about the 37) Fa rectangular cross section. Thus, any small cross-sectional area can be regarded as an element of a rectangular cross We start analyzing the shear ) of thin-walled open cross sections, and hence find 1(Q= sa | 8.38) wherein so= f * sa = [24a 62 z z ‘again is the first moment of the cross-sectional area below the arbitrary level ¢. 32 Shear Stresses 49 In the same manner, we can also treat thin-walled closed cross sections, Due to ‘the assumed symmetry with respect tothe zz plane, r must vanish on the 2 axis, ie, for y = 0. This allows us to divide the whole cross section into parts with vanishing shear stresses at both ends, Example 3.5: "profile shown in the figure: a b=h=200mm, 5 = 10mm, at §Q=100/3kN. i ‘Determine the shear stresses for the T= ke ae Solution: fi ‘We introduce the profile and the new axes ¢;, i = 1, 2,3, and determine the distance eto the centroid. = [a me fe ; I Fig. 38 Coordinates 6 erik 2h 4 TH Iyy = HH + 26h = an + Zan = Fans ‘To solve this problem, we use graphical integration method, starting withthe dis- tribution of the function 26(¢) along the different C, axes. Here, we have made Use of the fact that the total integral ofthe first moment ofthe cross-sectional area vanishes, i. f:s@e= [icsqacs f° 26(6)a6 =0. 3 : Thus instead of using the definition (3.39), we have 50 =- [sso 4 503. The Theory of Simple Beams I one /8 ‘we arrive at the distribution of the shear stress 7 along the different ¢; axes, with a ‘maximum value of r = 22.5 MPa at the centroid. A= 26(b-+h) = 56-10" mm? Less yet? mnt. J = Gan + bb = 5 Shh +36) = 648-10" mt. ) Bi says ee eee ame on | 50) sto) on(H-4 26) oo Fig 3.1 1 rs , “Thio-walled square cross “ De ‘ason| tion fonctions 4, and 3 of the shear stress 7 is obtained by multiplying $(¢) by the te funn} foe +3 = 4, eee esas netic cee es is equal to the vertical load Q=2%h. We now assume that the square cross section is a welded structure, with a long weld in the webs at z = 0. We further assume that due to some carelessness during the ‘Welding process one of the welds, the right one (say), breaks, and that the closed ‘cross section turns over to an open one. In the second part of this example, we 523. The Theory of Simple Beams 1 1@— gomn+anl bow? ' 30, fe gt Bon(h +40 Thin- wale Bonin |___—"#ron seton, anton 3 ‘are now interested in the influence of this change on the distribution of the shear stresses. The distribution of the function 6(¢) isthe same as before, but ue othe differ- tegration over ¢, however, we get a different distribution ‘ofthe first moment $(C), which is no longer symmetric with respect tothe 2 plane. It follows from these considerations thatthe distribution of the shear stress 7 hhas changed drastically compared with the solution ofthe closed cross section. The symmetry has been lost and the maximum value of the stress has increased by a factor of 2, i@ me { tn ieee EN Ty Ao B esate tt ! ress cae sl nee ees rn ee a Moreover, if we calculate the resultants of the shear stresses in the different regimes of ¢, with = 0.0169Q (5h + 126) = 1.0938Q oe = Ty= FZ AnMh+20) = 0.32000, ‘we have to realize that the horizontal stresses in the flanges no longer cancel one another. Instead, in addition to the resultant ofthe vertical stresses tion through which the resultant of the transverse shear forces always passes, re- 3.3: Shear Center of Thin-Walled Open Sections $3 dng moment about the x axis, which does not tng thatthe assumption that the beam axis to be modified. This inconsistency may help us to determine the {gardless of the direction of the transverse external forces. This point is called the shear center ofthe cross sect the locus of the shear centers ofthe cross sec- tion of a beam is called the elastic axis ofthe beam. Loads acting on a beam must pass through the elastic axis in order to produce simple bending of the beam with ‘no twisting. It's straightforward to show that the shear pple 3.6 must be located on the symmetry axis, that the moment of the different shear forces (see Fig. 3.13) Ta(y0 ~$) ~Tyh-271(vo+ 8) =0 istance y p from the x axis, such jth respect to this point vanishes | From this, we may calculate ae _ her vo= 3} (ret +rn-+78) = 242 — soem ‘The location of the shear center isnot alway obtainable. Ifthe cross section has an axis of symmetry the shear cente located on this axis. If the cross section has two axes of symmetry, the sh il coincide with the centroid. From the above results, we may conclude 1 Q.v ~ Quen = fr HOaCCaG 40) for the general case of transverse loading, where a() is the distance of the specific Point of the cross-sectional centerline from the elastic line of the bar, i.e. the axis Of the shear centers ofthe cross sections. Introducing the results of Section 3.2 (Eq. (8.38) - and its generalization to multiaxial loading), we finally arrive at 543, The Theory of Simple Beams I w= a “a(OSy(046 G41) o-Z in a(6)S.(0) a6. 3 If the shear force does not pass through the shear center, we end up with an additional twisting moment of the magnitude resultant shear force times the distance from the shear center. In the case of Example 3.6 this would mean an axkitional twisting moment of Mr =-Qu = -0504Q {kN}. [ c 6, Solution: ‘The distribution of the shear stresses along the profile of the cross section is com- puted according to Ea. (3.38) Example 37: For the thin-walled semicircular section shown in the figure, compute the shear stresses along the profile, and the coondi- nates ofthe shear center. 540) = [1080 a. For a thin-walled structure, with = const., we hhave dA = 5d¢, and thus for the moment of inertia 3 where use has been made of the transformation relations 2 = rin y and d¢ = rd. In the same way, we compute the first moment .39) of the first moment of the cross-sectional area will 5 tga [P50 = fess? eae = For, 5 so = [1osac Sia faie = br? con: : : 3.4 Influence of Distributed Loads 55 From this result, the shear stresses can be de- termined z 110) = 72 cosy. Ea) < "The distribution along the profile is shown in Sec © ‘the diagram, where also the direction of the shear flow t(¢) = 1(C)é has been sketched. ‘The resultant of the vertical components of this shear flow turns out to be r= fu eos ede = ee odp=Q, ‘other. Obviously, however, this shear flow has a resultant twisting moment about the z-axis, which must be equivalent to the moment produced by the transverse shear force Q aa [rae reall ones ‘From this relation, we compute the distance a of the shear center from the 2-axis ar Fn i a= =f a1273r ‘Alternatively, we may compute this value directly from Eq. (3.41) Bs ost Ses hctexgr ae aif a(C)Sy(6) d6 = 3.4 Influence of Distributed Loads __ In this section, we will discuss the influence of distributed loads acting on the outer ‘surface of a beam on the shear stresses ofthis beam. For the most simple case of plane bending of a rectangular cross section, the Toad may be given by functions 7, ps ating on the upper surface ofan element of, length dr. The equivalent distributed forces (and moments) n(x), g(r) and m(c), ‘respectively, acting on the beam axis are nz) = eb, m(z)=—peb 2, a2) =ab. G42) From the differential equation forthe esultants N(x), Q(z) and M(x), ic. from 56 3, The Theory of Simple Beams 1 7 h il Fig. 14 Element of beam with dis- bi tributed loads .43) these resultants can easily be determined. However, the question remains, whether such distributed forces will influence the shear stresses. 1. Influence of a distributed axial load pa differential equation Since a symmetric rectangul: considered, we ean make the same assumptions as before, name! 5). As before a small element of, length de and height dz may be cut out of the beam. Again integrating over the relevant surfaces, equilibrium of the forces inthe direction yields (Eq, 3.26) a a U ole. 1 bra | Pea :A 5.4L r(x, 2)bdr pare — { qi Shall emt of 8 JB ole, 2)bas {3 o+92asjbds Seewes” “mon 1(z)bda = [mete (G44) ‘The partial differential of Oo 8 { N(x) | M _14N aM = e-g{te+ “fees po) srom which, with Bq, 43) oe __ 1) 5 5 19(0) (oy). e 6.46) aA 34 Influence of Distributed Loads 57 Introducing this relation into (3.44), we find ni 4 1(2) = [Q(2) - mio | bas ‘The integral herein represents the zeroth moment of the shaded part of the cross- sectional area with respect to the y axis. For the rectangular cross section under consideration, afretG-R-e(-2) om “Thus, with (3.31), we get £ 3Q az\") pe f, 42 _(22)* = 2 9f-(2)]}-B fea) }] ow ‘The result shows that the additional term of the shear stress also varies parabolically with . The stress is zero when z = h/2, and ~ps when s = ~h/2, which coincides withthe load on the upper surface, because of O22 = 0s, ‘This result also remains valid when we leave rectangular cross sections, and deal G50) now allow normal stresses surfaces, equilibrium ofthe forces in = direction yields = frcapacs f frie) + z ar} bas ~ox.(2)bde =0, “from which 58 3. The Theory of Simple Beams 1 baad APS ote.2f? H il hapa Aq is i 7(@2)— des} pL tata , 4 ZA [2 (e+e aeybae 8 or 1 stor i mE Si Sorel eu. ‘The partial differential of hvtined from Ea, ae or _ 8 fal iO}. AQ Sy = Or” Ox ‘de Jb mat 7 O22 = % de pricier aa ee 4 3221 (2 [seny ee ; from which we finally get x 1238128 eee aan 2 Gala son mig sar Stresses 0. Fig. 316 Small element of a beam ‘with distribution of stresses G31) 3.52) 53) G54) 3.55) We realize that any distributed vertical load p. is associated with a normal stress ‘ze, and that this stress is of the order ofthis pressure. 34 Influence of Distributed Loads 59 ‘Again, this result also remains valid for any symmetric, but more general cross section. Example 3.8: A simply supported beam is subject to a horizontal load on top of the upper surface "as shown in the figure, Compute the sheat stresses of this beam, P= const. ail if Seer B EPP Wt rat aires hela fe ; Eaten eal , Solution: ‘The equivalent load with respect to the beam axis from Eq. (3.42) is n=prb, m=—pab*, With this information, we can solve the differential equations for the resultants (3.43), which - taking into account the boundary conditions Sl fis (ai sacs (la fama ks =); 60 3. The Theory of Simple Beams I If now a vertical load p, is applied to the above beam, we find a=mb > M(2)= Fp.ba(t-2), 1 Q(2) = $9.6 20) ‘Thus the (classical) shear stress ris determined from Ea. (3.17) with a maximum value of she Tous = Ps 5 at both supports. Fora long slender beam with 1/h > 1 this is much larger than the maximum value of normal stress ¢::, and thus =, is of negligible order. This, hhowever, is not the ease in the middle ofthe beam or, generally, for short beams. 3.5 Stresses in Non-prismatic Beams ‘The beams discussed in the preceding sections were assumed to be prismatic, ic. Fig, 319 Non-prismatic beam For the shear stresses, however, the formula (3.32) derived for prismatic bars is no longer adequate. Instead, we must derive a new relationship that incorporates the effec of the changing height (and width) ofthe beam. ‘Considering frst plane bending of rectangular eross sections, we may start with the same assumptions that have been made for calculating the shear stresses of con- (3.25) has to be replaced by (3.34) again, 35 Suesses in Non-prismatic Beams 6 In the same way as we did for prismatic beams, we cut a small element of length ~ dr out of the non-prismatic beam (Fig. 3.20) with 2tana = Ai2) _ ae ~¥e) + i tea eg? Joba: + 2/9 obdz}ae BEI lement of non prametc beam "Integrating over the relevant surfaces, the equilibrium of the forces in direction yields ousl2) = 5 { I wa} 56) persis i ae oul) = te (ie asf was sep a a} @s7 and finally no SPH D]] oe where A(2,2) = / sey re eee Haars as) ae the zeroth and the first moment, respectively, ofthe shaded portion of the cross- sectional area with respect to the y axis, which are now functions of + and 2. For a rectangular cross section, we find ate) = jac {1- wm} Syer2) = preyate){i~ (25) } ee as, (2) et (3) -8b9G) = and thus cntea)= 329(1- (3) } fF - aS p-2 G) Toe We 222) = Lone), G61) with: also upon the normal force V and the bending moment 1M, and the rate of change of A with respect to, Example 3.9: As a specific example, we investigate the distribution of shear stresses for the can- tilever beam of rectangular cross section shown in Fig. 3.19. The beam has heights hha and hy = 2hq at its ends, and a uniform taper Nz) = ha (147), Solution: We find N=0, Q=-F M=-Fr. ‘Thus, at both ends, and at the middle of the beam (:r = 1/2), the following distribu- tions are obtained W(@) =2tana = * eet: ma 3 {t-(E) } ral: 0, --}£--Z, s2th pec Fig. 321 Suess in a non-prisms 22 beam aaa The different distributions are plotted in Fig, 3.21. At the let end, the moment A is zero, and hence we find the same distribution ofthe shear stress asin a prismatic ‘beam, withthe maximum value atthe neutral axis (2 = 0) At the middle of the beam, we obtain the interesting result that the shear stress is uniformly distributed over the height of the beam. At the right end of the beam, ‘again a parabolic distribution of the shear stress is obtained. Note, however, that the maximum value ne Ieeamsl = 3 occurs at the outer edges of the beam. a If n addition tothe preceding analysis a changing width is examined, a similar ‘calculation leads us to additional information about shear stress xy, namely Gy(2,2) = Ony( 02 (363) "while the relation foro remains unaltered, and tang = 3¥@) 3.64) "expresses the rate of change of the width with respect to 2. 3.6 Deflections of Beams ‘The deflections ofa loaded beam are analyzed by considering the deflections which Bere scsoo teauns paste Reuse ea opto sos waceet hes 64 3. The Theory of Simple Beams I forces on the deflection is neglected altogether. We will come back to this point in ‘Chapter 6. In the following discussion it will be assumed thatthe loads on the beam act in the 2 plane, which contains one of the principal axes of inertia ofthe cross section, ‘and that the bending moment vector is normal to this plane. Inthe deflected state, the axis ofthe beam will then bea plane curve inthe zz plane. 2s the deflections of a beam are small compared to its span, ‘may be neglected against unity. Then Eq. (3.66) assumes G4) ‘where the increase in temperature is given as = +H: 3.69) Defining now Nt Ba f eas, M?= za f 204A, (3.70) Is fi wwe arrive at o. BJ _ ge. N=BAcp-N', M=>-M’, am wherein N° = BAab, M’ = EJab, G72) describe the contributions of the temperature field. Introducing these informations into Eqs. (3.65) and (3.67), we finally get the differential equations EAu(2) = N(z) +N* BJu!"(2) =—M(2)~M?. 73) 36 Deflections of Beams 65, Differentiating these equations with respect to and then substituting Eqs. (3.43), yields of boundary conditions are given in Tab! 74 system | kinematical conditions ge [eee wo gfl- v=o N=0; M=0 je | uno w=0 a Q=0 | | uae w=0,w=0] - - t - w=0,w=0| N=0 - eS N=0; Q=0,M=0 ‘Example 3.10: ‘The frame has a constant flexural rigidity E.J. Compute the deflection curves w (1) |. the reactions as well asthe nor- 66 3, The Theory of Simple Beams I Solution: We divide the frame into two regimes and introduce for each a coordinate system as shown inthe diagram. ‘The system is statically indeterminate of degree one. Thus, since the function of the bending moment isa priori not known, we start from Eq. (3.74), neglecting here for the sake of simplicity the influence of the normal forces on the deflections. We therefore have the differential equations ) =a, Ginregime 1) 0, Gnregime 2) and pertinent boundary conditions w1(0) = u4(0) =0, Q(t) =-BJwf"() =0, ‘w,(0) =ta(!) = 0, Ma(0) = -BJwi(0) =0, and transition conditions wh (1) = wa (2), Mi(l) = —Ma((). ‘These are all together eight conditions to determine the 2 x 4 = 8 integration con- stants ofthe problem. ‘The solutions ofthe above differential equations are 1 i 1 Edw, = Fait Zarit Zari ten +e, Edun = Lesa + Seon +er0a + c4 From the boundary and transition conditions, we compute @=4=%0=%=0, and ea) ‘ae ah 5 bys Poll baidalass i = Fal, = — Fal, ‘Thus we arrive at the deflection curves Bjw joy = © at (20} — Bz + 90" 5 Edw = 5 2 alr, +91) , jas = ig 2} hw. = tz (3-P). Having solved the differential equations for the deflections, we are now in the position to give the diagrams for the stress resultants, the system is stati- cally indeterminate. We therefore refer to Eqs. ‘compute the functions of the resultants as deriv 7 P ia a aa | Ny @ Mw 1 aw 3.7 Exercises to Chapter3. 67 8 op = ser 5” ‘We mention in passing that without recourse to any theory of statically inde- terminate systems, we nevertheless were able to solve a static system, Thus Eqs. (3.74) moreover deliver a frst method for solv terminate systems. _ 3.7 Exercises to ‘Chapter 3 Problem 3.1: |For the following cross sections, compute the principal moments of inertia and planes: Problem 3.2: Problem 3.3: For the cantilever beam shown below and for a = b/4, determine using the maxi- ‘mum normal stress criterion the equivalent stress at = 0. 68 3. The Theory of Simple Beams 1 F 10F = /— # Lb ia 108 —— es Problem 3.4: Determine the maximum compressive and eee tensile stresses in the Z.section ofthe beam 4 Problem 3.5: ‘A cantilever beam with the built-up section of Problem 3.1b is at its free end (« = 1) subjected to a normal (tensile) force F acting on top of the upper vertical leg. Determine the extreme values of the normal stresses. Problem 3.6: For the following thin-walled cross sections, compute the shear stress distribution, ‘and the maximum shear stress, Problem 3.7: For the following thin-walled cross sections, compute the shear stress distribution, ‘the maximum shear stress, and locate the shear center. Mey Hee 37 Exercises to Chapter’3 69) 38: For the frames below with constant flexural rigidity J, compute the deflection ‘curves wi (11) and wa(sr2). 70 3, The Theory of Simple Beams 1 e temperature at the top of the bottom is 72. Deter- 2 —____., and the deflection flexural rigidity constant. Problem 3.11: A frame is subjected to a vertical force F at the point ry = 1's shown in the diagram. Compute the deflee- tion curves tw (11) and wa(r2). Note that the flexural rigidity of the hor- ‘zontal section of the frame is two times that of the angled-section as shown inthe diagram. Problem 3.12: ‘The flexural rigidity of the cantilever beam shown in the figure is a function of coor- dinate 2 The length of the beam is f, and it is subjected to a linearly varying load of intensity q(x) = ax + 8. For eu(a)=K(S2°+ 22"), determine the deflection curve of the beam, the angle of rotation, and the deflection at the free end of the beam. 4, Torsion of Prismatic Bars 4.1 Solid Cross Sections Torsion of a uniform bar of homogencous isotropic material means twisting defor- ‘mation produced by equal and opposite twisting moments applied tothe ends ofthe bar. “The problem of uniform torsion for bars of any cross section is solved by the St-Venantsemi-inverse method, Certain features of the solution are at frst assumed, then possible to satisfy all the equations and boundary condi- tions validates the assumptions. Tis assumed first thatthe eross sections rotate about the « axis, the (small) angle of being dz, without distortion in their own planes, and where 0 is | the angle of twist pr unit length, or shor, the twist." The displacement components ty sus in the plane then correspond to a small rotation x of a rigid lamina, and are uy =—022, = Ory. ay) ‘The axial, or warping, displacement u. is taken as a function of y and 2, uz = 00(y,2), (42) where v)(y, z) is the warping function, “The six strain components follow from these displacements as Ou dota be wollte tee FE =O, ty = GE =O, t= G0 (43) and _ 2 fav mH} ce f= 3 {3 +i}. a) Lf dus | Ou, 2 [ote oy 2 en = 3 {Me +88} <0 T We note that according to the assumptions of St.Venant’s theory this twist has to be a con- stant For non-uniform torsion, where the rotation ofan element of length dis described by dip = (2) de, we refer to Section 4.4. 72 4, Torsion of Prismatic Bars ‘These give, with Hooke’ law, the stress components, Oe = Oyy = Fre = ys =0 as) on n00{ 2}, =ce{ ae ~oo {Fe +s}. From the three equations of equilibrium (Eqs. 1.61 and 1.62), two are identically satisfied, and the third reduces to 0,2, Ooze _ aaa a7 provided, we neglect acceleration and body forces. ‘The problem of torsion may now be treated in different ways: 46) 1. The first system is obtained, when we introduce Eqs. (4.6) into the equilibrium equation (4.7) Fy, Pw aye oh. 9, 4. = Gat ot as which gives an equation for the warping function. “The boundary conditions ofthe problem are such that the resultant shear stress at the boundary must be directed along the boundary curve, since the surface is assumed tobe stress free, ie. 49) where the boundary it yas), == (4.10) with are length s measured from some fixed point. Using Eqs. (4.6), the boundary condition may be expressed in terms of the warping function as a faroect- $0 a 2. Forthe second way, we ‘(y,2) from the two equations in (4.6) to obtain Pome _ es 369, (4.12) By It follows now that zy and os may be expressed in terms of a single function ‘P(y, 2), Prandtl’ stress function, as ar ar O04 =2005 on = 2605. 4.13) This function must satisfy 4.1 Solid Cross Sections. 73, ar= 5455 =-1 (4.14) ee ey 15) "This means that 7’ must be constant around the entire closed boundary curve, with different constants at different curves, e.g. if there are holes. One of these constants may be arbitrarily assigned as zero, such that 4.16) Itis advantageous to consider, in general terms, the evaluation ofthe torque Mr. and the warping displacement i: The torque Mr is given by Mr= I (2:4 — 2y3)4A a7) 4 the integral over the material cross section (excluding holes). This can be expressed | in terms of the stress function 7’ as or or p= 260 f {Fo y+ Fs} aydz (a8) Finally, after integrating by ars, the total toque is given by Mp=4G0 [7(,2)04, 19) 4 which is twice the volume under the T'(y, =) surface multiplied by the factor 29. From this relation, we can introduce the polar moment of inertia Jy as Jr=4[TUn3)aa 420 4 ‘The increment of 1(y, =) corresponding to any arc element ds ‘in the material section is os BG 4.211 d= Fay + as, 21 i.e. from Eq. (4.6) « °. dy = {2% + zhay+ {= yh de (ederlti) ms = Gg Cova + 2s dz) + (2dy — ydz). 744. Torsion of Prismatic Bars ‘The sum of these increments forall arc clements of a closed curve lying entirely in the material must vanish to avoid discontinuity in y. Thus, we find frss=2G0A0, (423) é the shear-circulation theorem fora closed circle C, where Ais the area enclosed by Cand 7, is the shear component in the direction of s.. Example 4.1: Determine the stresses and the warping of a solid elliptical cross section. Solution: ‘The problem is solved (indirectly) by finding the stress function which satisfies within the ellipse gt the differential equation (4.14), and on the ellipse satisfies the boundary condition T =0. The function is 2? 2 A ise este (URS roar Tam{S+3 1}, "ata “The stress components ae (Eq. 4 2 ny = 2G OM, Oe = ‘The shear stress 7 is the resultant of oz, and a, and so aes ate 7 =4GOm ‘Stresses in an elliptical eross section ‘on the boundary at the ends of the minor axis, that is, atthe points nearest the axis ‘of the torsional rotations. @ vac aewlt |The lines of constant values of the warping are shown in Fig. 4.2. ‘The torque is, from Eq. (4.19) ‘Warping of an elliptical cross section @+P From these results, we ean deduce the following relations a oies, te We ae By wet oe Oe Mr _ 7 att Bam ip a a Corresponding values ofthe polar moment of inertia Jr and the section modulus Wr may be determined for cross sections of different kinds: 1. Equilateral triangle Let the boundary of a torsion member be an equilateral triangle with h = /3a/2. aba Hit ‘Equilateral triangle Proceeding as for the elliptical cross section, we find ely Depp Lga T= Fer Wr a50% Sa ee o =e, Ba, 424) 76 4. Torsion of Prismatic Bars 2. Narrow rectangular cross section ‘Consider a bar subjected to torsion. Let the cross section of the bar be a solid rect- angle with width 6 and depth h, where b < h, From the different analogies (e.g. ‘the soap-film analogy, originally proposed by L. Prandil.),? we may conclude that except for the region near = = +h/2 the stess components oy and cs are ap- bb | @ a i | see t Fig. js Narrow rectangle proximately independent of =, and Oxy =0, Ges ¥ ry) = 2G0y. (425) ‘Thus, from Eq. (426) 27 Its, which are valid only give a good approx- tend zones, We may Wr=aghP, r= (428) Analogies exist where physically diferent problems have similar mathematical descip- tions. In this ese solutions. or experimental findings - from one problem may be tans- fered tothe other analogous - problem. The most known analogy tthe torsion problem ‘of prismatic bars stat ofa membrane (soap film) fxed on a closed boundary, having the same shape asthe eros section of the torsion bar, where pressure is applied to onesie of| the membrane. We therefore refer eg othe textbook of Borst, Schmidt & Sidebotom, ‘Advanced Mechanics of Maverals, Sth Editon, John Wiley & Sons, NY. et. 1993. 4.2 Thin-Walled Closed Cross Sections. 77 ‘a | 0.63 | 0.69 | 0.74 | 0.80 | a5 | 0.92 8 || 0.42 | 059 | 069 [ 0.79 | asa | 092 ‘Table 4.1. Correction factors a and 3 woe] 1 [isl 2] 3 ]4]8 lo 1 1 4.2 Thin-Walled Closed Cross Sections In the preceding section, we have discussed torsion of prismatic bars with solid This allows us to assume G2, =0, 92¢=7(6) (429) forthe shear stresses, and u=ule,¢) 430) 431) ‘we obtain with (4.29); 10) = 11980). (432) 78 4, Torsion of Prismatic Bars Qe {r+ Ge(r8) dc} de From the equilibrium (in axial direction) of a small element cut from the thin-walled tube (Fig. 4.6), we see that t= 1(6)6(6) = const, (43) stresses ose. Thus, the largest shear stress occurs where vice versa, Of course, if the thickness is uniform, then around the tube. sr to relate the shear flow to the torque M'r acting on the tube, consider an element of length d¢ in the cross section: Fig. 4.6 Element of thin-walled tube aed Tro ~y v 71 S|. sehen re j cae sat |. 47 HS ab es aaa! ‘The (otal shear force acting on the element is ¢d¢, and the moment of this force bout any point 0 is 434) in which a(¢) is the distance from 0 to the tangent tothe centerline. The total torque then is Mr=t f (6) a6 = 2Amt, (435) ‘where the integral represents double the area Ay, enclosed by the centerline of the tube. From this equation, we find t= r100) = HE 436 and finally, 4.3 Thin-Walled Open Sections 79) Mr i Fat = Fg Wir = 2Amb(C)nin (437) Bredt’s first formal. “To determine a relation between torque Mr and twist 0, we star from assump- tion (430), and e() = 010), ue = Poel). 438) With these displacements, we find ex) = $f $2 +000}, 439) and thus from Hooke’s law uc(6) = 110 = 20a 4.40) ‘Integrating this expression over the entire length of the centerline yields froa=f f au. +0 f each (at) “The first integral of the right hand side vanishes (continuity of u..), and thus with Eq, (4.36), we finally arrive at - Bredt’s second formula. fin Eq. (4.41) the integrals are not taken over the entire length of the centerline, ‘eg forthe first integral of the right hand side fou = up(C) ~ u2(0) = H{H(C) — ¥O)}. (4.43) (6) = 00) + 24m fe a if a(c) ag (aad) 5G) from which the distribution of the warping along the centerline may be calculated. 80 4, Torsion of Prismatic Bars ~%. 50) GF ‘Thin-valled open section 4.3 Thin-Walled Open Sections ‘Expressions for thin-walled open sections are based on formulas for the torsion of the thin rectangle. For sufficiently thin-walled sections, good approximate formulas across the thickness 46(G) as ina thin rectangle of the same thickness. From ‘make the following assumptions as before cf. Eqs. (4.29) @ om=0, Gi) w= ulr,¢). Furthermore, we take oz¢ from the thin rectangle as a linearly varying function of (4.45) Finally, we assume : 1 famarend 3 w sang [MOK=15 EA 4.46) wherein 7 is a form factor (cf. Table 4.2). Profile] L_| 1 | C | I 7 pe [oe fs Table 42 Form facory From these assumptions, similar to the results before, we find — Mr on 2, aan a = ae t= ME, Wr= 2. 4s) ‘Since according to assumption (ii) the shear low vanishes, we find for the warping function 423 Thin-Walled Open Sections 81 0 =¥0- fatwa. a) : Ge age feck asec ofl om co ‘The different formulas to describe the torsion of prismatic bars are summarized intiied3. Example 4.2: Determine the polar moment of inertia Jr and the section modulus Wy, as well as the warping function ¥(¢) of the hollow square cross section of Example 36. ‘Compare the different results for closed and open cross sections. Solution: ‘The dimensions are taken from Example 3.6 as b=50, h=90, 6=2 [cm] and thus we find Am = bh = 45-108 em? WF ec aaa = 18-10% cm? 264m) = 10 Hn OH = Spa 8 57857. 10% emt for the closed eross section, and dp =5 zo +h) = 746.67 em* Wr= = = 373.33 cm* for the open cross section, where the form factory has been introduced as +) = 1 ‘Comparing now these results, we get the following ratios Which means that the twist @ of the open cross section increases by a factor of ‘71487, and the maximum shear stress Tax By factor of 48,21, compared with the respective quantities ofthe closed section. “The distribution ofthe warping of the closed cross section is calculated from Eq. 44) 82 4, Torsion of Prismatic Bars 43 Thin-Walled Open Sections 83 Mt j , bara ‘Twist: a= BE Rovation e= foe)ar+ (0) i es Shear stress: Fring = Hel Jr Wr ‘Stresses, form factors , y vc) — #00) Sod rou tas i nd! xd Mr, q at cc op = hap Peso © ee ¢ Ge [Bhs Relative warping #(6) ~ v(0) AK: PY ¢ cetee ltd Belt uN = 5 vo=wor+ rex a [oo ey a pS |e Pos “The free parameter 1(0) herein is determined from Eq. (4.52) 7 is Wr agen | atin [a | 068] ora] ons] 092] 1 flv vo) a+ f «=o 8 || 0.42) 0.69) 0.84) 0.92) 1 a Tie- vale asl secons hi : a = 0) = Ab sat asent. Ge) je [|e petit ioe %6) ‘TES i < wo = #0) Jaco, Cy tua, | Tae ; 4 ‘where a(¢) is now measured from the shear center of the cross section with ‘Bbh + 2b wHe w= > = 59.38 cm. a [ “Ls t+h bad Dh+3b ‘Thin-walled open sections 1 dri) IMr| <9 Fras? | Eads | trl) EEL Che ap i & rfarox wpa we. | (Pome CTL Tt Tr 7Ehen jem | [ot] 29] 2.29) 23] $o0+ $f — 0 +8) 2h ¢ yeales) (6) — ¥(0) | bp 4e 3Q6-wl leo) Fig. 4.10. Relative warping ¥(¢) ~ (0) Table 4.3 Torsion of prismatic bars 84 4, Torsion of Prismatic Bars ‘The free parameter 1(0) herein is determined from Eq. (4.53) [181 - wo) +90) [aco = L ¥(0) = bh = Introducing these into the above distributions, we finally get the warp- ing functions as ai 703.18 L_|:+s000 to / 2148 703.18 closed and (b) open eross section ‘Moreover, from the above result, we can calculate the relative displacement of both ends of the open cross section Mr Au, = 0p = Bese = 1205-7 fem Example 4.3: Determine for the open cross section of Example 3.6 the additional shear stresses due to torsion. Solution: ‘The shear force @ is acting in the centroid of the cross section, Thus the equivalent force system acting in the shear center of Q and the torque Mr = -Qup = -59.38Q _[kNem|. From Eg, (4.48), we find Mel 5 _ eG a = 80.0637 is Tima Mr ‘The distribution of the shear stress due to the shear force @ is obtained by multiply- ing the function $(¢) of Fig. 3.12 with the factor 44 Influence of Restrained Warping 85 i 9 60 J6 ~~ §h2(h + 3b)’ with the maximum value tonlg = 3 224% = 2bhh+3b ‘Itturns out that the maximum shear stress — in the center of the we ‘contributions. The first contribution is due to the shear force, ane = 0.0066Q = soot ee asa 244 sr ea at eco , a Consider a hollow rectangular cross sec- a tion subject to torsion. The dimensions of T this eros section are b, fh, and 61 and 6 f Provided there exists a eross section with- +] J ‘out warping, determine te specifi dimen- sions ofthis cross section, Solution: ‘The warping of a closed cross section is described by Eq. (4.44) as aC Se Ec 10) = 10) + 24m fe faa om ° hdd ¥(¢) = (0) = ior bah | - “a(coac =}. Since this expression has to vanish for every G, ie. every point ofthe centerline, it follows that D552 aed adic Every closed cross section obeying the relation h/b = 51/62 is free of warping. 4.4 Influence of Restrained Warping In the uniform torsion of a uniform bar (St. Venant’s theory), the originally plane ‘eross sections become warped, the warping displacement w, being nonzero (unless the section is circular) and such that the cross section becomes a (slightly) curved 86 4. Torsion of Prismatic Bars surface. This warping is the same for all cross sections and is proportional to the ‘owist 9, of, equivalently, o the torque Mp. The St. Venant solution prevails only It needs emendation if the warping is completely Be = oH), 450) I the warping of the unrestrained torsion (Eqs. 4.44, and , de. ‘We note that here, in contrast to what has been adopted in Section 4.1, the gradients in Eq, (4.50) have been derived for a small element of length de ‘The free parameter 1(0) of function 1(¢) may be determined from the condition J[voaa-o, asp ie. fiooqac=0 4.52) for closed sections, and \a(@) ac = 0 (453) Furthermore, we keep assumption (4.29), ie. 29 = 0. it immediately follows that Son =0 454) On = 0(2,6) = Eee = EV'U(C) (4.55) 44 Influence of Restrained Warping Fig. 4.12. Element of a thin-walled profile and, with (4.55) or " ca —B8(G)H(Q)0" (x) 44.1 Closed profiles ‘The resulting torque is (4.34) (4.58) (4.59) (4.60) (4.61) Of the first integral then allows the introduction of Eq, (4.58), and thus, we finally arrive at ‘Mr = GJr0(e) — BCr0" (a) = Mj + Mj with Or = f VOB A, the warping resistance, ‘The differential equation yolw® w_ Gk Pie ere eral hhas the general solution (4.62) (4.63) (4.64) 88 4, Torsion of Prismatic Bars (a) = €1 sinh Ax + cp cosh Ae + Op 4.65) here @ is the particular solution, and the constants ¢) and c2 are determined from the boundary conditions. Typical boundary conditions are, e.. 9 =0. fixed end W=0 freeend. ‘When 0 (2) is known, the stress components are determined by Eqs. (4.55), (4.58), ‘and the formulas ofthe St. Venant theory. ‘We realize thatthe torque Mr in Eq. (4.62) has been split into two | the first term describes the problem of pure torsion with free warping (St. Venant’s theory) whereas the second term is related with the restraining of warping. 442 Open profiles ‘This is also true for open profiles, My = Mj+ Mj", (4.66) where now Mj. = GJr0(2) (467) is related with a vanishing shear flow ¢* = 0, and Jr is given by Eq. (4.46). The necessary to prevent the cross section from warping, (4.68) imilar procedure, we can show that the above differential equation (4.64) is also valid for thin-walled open sections, where J'r is given by Eq, (4.46), and the ‘warping resistance may be calculated from Cr= f MOKA (4.69) Example 4.5: ‘Acanilcver beats soblecttoatonme Mp. Warping is prevented at one end : Mr. (x = 0). Determine the twist and the = nal anil stresses Ose. i— Solution: Ate = La torque Mr is applied, and the axial stress oj. is zero. Here Mr is constant, and the general solution (Eq. 4.65) becomes. 44 Influence of Restrained Warping §9 = HE (+ sinh Ax + enconh Ae), wa Se, ‘The end conditions are wy =O at 2 = 0, and cre = 0 at = I. By Eq, (4.56), and the equation u, = Oy, these require 8(0)=0 and v'(t)=0, siving =tanhN, 2 =—-1. ‘The rotation of cross section is = fose= Me fey} pe [osama (es xem Mens 1) aaa] | Then, from Eq. (4.55), we find Oxe(2,6) = Ev0! = ME yey(¢){tanh Mcosh Az — sinh Ax}. Gh Example 4.6: A.cantilever beams subject toa uniformly y distributed torsional load mr. Warping is mE ‘prevented at x = 0, Determine the twist 3 = nd the axial stresses. :—_ mp =15kNm/m, = 10m. Solution: ‘The load mis per unit length, and the torque at point 2 is Mr = mr(t—2). ‘The end conditions are u, = Oat 2 = 0,and x2 = 0 at x =I, giving 0(0) =0 and ‘The solution of Ea. (4.65) 'ying these conditions is, mrt 0 = Gy (—F teasinh Ae + excosh Az), with, L+Msinbl Mee Aly heh yee: From Eq, (4.55), we find @a(2,6) = Ge-BO(O)(—1 + Mer cosh Ar ~ Atsinh Ax}. and the rotation of erss sections is a= 904. Torsion of Prismatic Bars 2 Introducing now the cross-sectional quantities of Examples 34 and 4.2, we frst determine the magnitude of \. From Ea. (463), the warping resistance ofa closed cross section is Cr= f “Thus using the distribution ofthe warping function, we obtain after integration Cr = 19.286- 10° em®, and from Eq. (4.64)2 sr +r Q = 1.074-10-Fem™? + M= 1074-102, ‘where Poisson's ratio has been introduced as v = 0.3. With this information, we are able to describe the behaviour of the cantilever beam. The maximum value of the Sat | a w= = 1.154.107? em™® ue Le all | Fig. 4.13 ‘Twist and axial stresses forthe closed cross section normal stress is obtained at = 0 = 27 Ey — Ica = £23: 2x(0) = ze BV(C(M — leno = £23.05 MPa ‘with the maximum value of the warping function 1(6) at ¢ = 0 (see Example 4.2). “These values are now compared with the results forthe open cross section. Using the warping funetion #(¢), we obtain 45 Exercises to Chapter 91 ‘The maximum value of the normal stress is obtained at = 0 Ox2(0) = 1587.16 MPa, ‘which would be far too high ina real structure, and with the maximum value of the ‘warping function y9(¢) at ¢ = 0 (see Example 4.2). The twist attains its maximum value asymptotically at x = 1 | differential equation ( the dashed lines in Fig. 4.13) becomes predominant, \with deviations according to the boundary conditions only inthe small regimes close tothe boundaries. With decreasing XI, these ‘disturbances’ of the particular solution increase until the whole solution is predominated by the boundary conditions. 4.5 Exercises to Chapter 4 Problem 4.1: Each of the following thin-walled closed cross sections is subjected to a torque M. ‘Compute the maximum shear stress and the shear stress distribution. Problem 4.2: For each ofthe following thin-walled open cross sections subjected to a torque M, ‘compute the maximum shear stress and the shear stress distribution, © Problem 4.3: A thin-walled closed cross section is sub- stress and the shear stress distribution. Problem 4.4: ‘The thin-walled open cross section is sub- 15cm, Q = 400KN, and M = 18 EN, compute |. the shear stress distribution across the 45 Exercises to Chapter 4 93, Problem 4.5: ‘A circular tube and a square tube are builtin at one end and subjected to the same torque at the other end. Both tubes have the same length, same wall thickness, same cross-sectional area, and are constructed of the same material. What are the ratios of their shear stresses and angle of twist? (Disregard te effects of stress concentrations. at the comer of the square tube.) OD Se a ener er For each | side as a length ofa, and d) i a rectangle with dimensions a x a/2. 1=4m, a= 30cm, My = 100 kNm, G = 8.1104 Nimm?. 4. Torsion of Prismatic Bars 5. Curved Beams 5.1 General, Statics Development of the theory of bending of beams was based on the assumption that, ‘before deformation, all fibers of a small beam element have the same length dr. 6.1) ‘where a new coordinate s has been introduced in the direction of the curved beam axis, such that z=a(s), 2=2(6), ($2) and n(s),q(s) and m(s) are the distributed forces and moments, respectively, acting ‘on the curved beam axis. As shown in Fig. 5.1, R(3) denotes the radius of curvature ‘at any points of the beam axis, Moreover, withthe transformation ds = R(s) dg, (83) 96 5, Curved Beams: ‘we may introduce an alternative description of Eqs. which sometimes is of advantage. We note that in the 1/R these differential equations take the same form as Eqs. replaced by 2. It will be assumed here that the axis of the beam is functions of angle ¢, vanishing curvature 43), where then s is considered, the following stress re N=N(Q), Q=Q(s), M= In addition to these loads, the increase of a temperature field @ is also taken into ‘consideration ‘To describe the stresses and strains atthe cross section, and the displacements of the beam axis, a second coordinate ¢ is introduced perpendicular to s and directed as the radii of the beam, stch thatthe deflections w and w are the components ofthe displacement vector ofthe beam axis in the ¢ and ¢ directions, respectively. ‘The elementary theory of curved beams is again based on the assumption that plane cross sections remain plane during deformation, Possible effects of transverse shearing forces on the deformations of the beam are thus disregarded (Bernoulli's hypothesis). 5.2 Large Curvature ‘As in the theory of straight beams, we further assume that (see Eq. 3.3) bending, the cross section will rotate with respect to its orig inal position by the angle \dp, and the different fibers undergo different exten- Fig.82- Undeformed (a) and deformed (b) beam element sions/compressions 4 di, relative to thei distance ¢ from the beam axis Adl = co(s)ds-+GAdg, 66) 52 Large Curvature 97 where Ads _ du(s) , u/s) dae) ila? teRIn) 67) {is the extensional strain ofthe beam axis. Thus the strain ¢ 44 = ¢ located a distance ¢ from the axis is Ad, Aa _ £0(6) +6502 €0(s) = ea: amps 68) aa and the change of curvature of the beam axis is ese ae a. (s) = (69) _ Ade _ sols) __ ule) (ai) — st) a Rd? *as\R@) RR? where (since 9 <1) ¢o is neglected against unity, and d= ds+Cdy, dp= 6.10) Ris) ‘Moreover, the small angle A dip is described by the small change of the slope of the beam axis dy af ap =—a(S2) + (4). iy Inodcing these esting. (5.8), we fin aive a Rx(s) #(8,€) = eo(s) + (3.12) R+¢ which is comparable with Eq. (3.5) for the straight beam. In accordance with Hooke’s law, the corresponding then Ri o(s,0)=B fens +o ¢ (13) ‘The stress resultants N and M are defined as Nt) = foscaa, Meo) = [cotscyaa. 6.14) 4 4 “Thus we find Neo) = {ow M(s) = n(s) BJ*(s), ‘with the usual definition of the area, and a modified moment of inertia —n() 2) Ona}: (a 98 5. Curved Beams re)= fas 6.16) ‘and where we further have made use of the Fact that the first moment of the area of the cross section with respect to the y axis vanishes fora system of principal axes. 6.17) RG) + ‘The distribution of the normal stress over the depth ofthe eross section is no longer a hyperbolic law; aros the wid ofthe eross section, they are 5.18) (5.19) If the normal stresses are known, the shear stresses may be calculated in the usual ‘manner (see Section 3.2), For a rectangular cross section we get as 7(8,6) = (5.20) Example 5.1: Determine the normal ste critical section may be appr ‘h/2, the inner radius ofthe hook. 1¢ hook subject to a vertical load F. The ‘a rectangular cross section b/h, with Ry = Solution: ‘We first calculate the resultants acting in the beam axis N=F, M=-FR, where R = Ry + h/2=h, and Ry = Rj +h, The modified moment of inertia is ren [o Sh ac = wofn Re — T= on 1). “iva 54 Deffections of Curved Beams 99 Introducing this into Eq, (5.18), we find gs uty ier } Lanois "A~ TART R+CS ~~ AQn3=1) he F ee @ ~ A” | -8,38 outer radius (5) ‘The numbers in the brackets designate the respective factors which would be ob- tained from the straight beam theory. 5.3 Small Curvature Ifthe depth of the beam is -ompared to the radius of initial curvature, then joing equations. The neutral axis passes N= 0),and, hence, for the normal and ‘beams, Thus, in curved beams having a small depth, the stresses can be calculated as though the beam were straight. 5.4 Deflections of Curved Beams In the same manner as for straight beams, we will utilize the relations between the change of curvature (s) and the strain of the beam axis ¢ o(s), and the displace- ‘ments w and w to develop the differential equations for these displacements. From Eqs. (5.7) and (5.9) we find 621) (6.22) 1005, Curved Beams where now the prime designates differentiation with respect to s, and where more~ over the influence of a uniform heating is taken into consideration. Introducing Eq. (5.21) into Eq, (5.22), we get 6.23) 6.24) _ Me), 918) EJ*(s) Ris) © ‘These are two coupled differential equations for the displacements w and w, which ‘can be decoupled for circular arches, ie. if R= const. Example 5.2: For the clamped circular arch subjected to 4 ‘a constant radial load q as shown in the A figure, compute the vertical and horizontal deflections of point A. Solution: ‘The systems statically determinate, We first de- termine the reactions at the clamp to be a H =-Q(0)=4R, V =-N()=aR, M = M(0) =4R?. From Eqs. (5.1), we find the differential equa- tion Qe) +A) =9, where we have made use of Eq. (5.3), and where here the prime stands for differen- tiation with respect to the angle p. The solution of this equation is Qlv) = ersing + cr eo8y. With this solution, we find forthe additional resultants N(e) = Qe) ~ aR = er cosy — casing aR, M(e) = [ Qe)Rdg = ~cReosy +eRsing + es. With the above initial conditions, we arrive at 5.4 Deflections of Curved Beams 101, <0, @=—aR, o=al?, sod thus N(p) = ~aR(1 -siny), M(p) = aR2(1 ~sing). Introducing this result into Eqs. (5.24), the differential equations for the deflec- tions rad w(p) + ule) = gy (MLE) + RN) =0 Me) p2_ GR yg ay (1~sing) whe) +u(g) = - MO) 2 = - ere we have assumed the curvature ofthe beam to be small ‘The solution of the second differential equation consists of two parts w(g) = wale) + ply), hee un(e) = casing +esconp eee uy(e) =- 25 e+ pease) is a special solution ofthe non-homogencous equation. Thus we find SS oc 27a w(g) = easing + cosy — 2 (2+ peasy) ‘The constants 4, es may be determined from the kinematical conditions ‘u(0) = w'(0) =0 as at, ee Boe oes Sar With these informations we finally arrive at alt anon sep sgeniy= we) = 2 Going + 200m — peony —2) [Now from the first differential equation, we compute ae esate Sein (el [eerie = Sy Roose — 2sing +29 + esing) +60. ‘Again, from the kinematical condition u(0) = 0 follows =a =- FF 102 5. Curved Beams and ule) = 2 omg 2sing +294 psing 2) “Thus the deflections of point A are dwn =— Fak, Bua = (Fx~2)aRe 5.5 Exercises to Chapter 5 Problem 5.1: ‘The semi-circular beams have flexural rigidity J, axial rigidity A and are loaded load the vertical and horizontal displacements, the curve up) for the displacement {n tangential direction and the equation w(p) for the displacement in radial direc Problem 5.2: ‘The circular beams have flexural rigidity EJ, axial shown in the diagrams. Determine in each case for load the vertical and horizontal displacements, the curv in tangential direction and the equation w() for the dis tion, Assume in each case that the curvature of the beam DM ' gies Ke 6. Simple Beams II: Energy Principles 6.1 Reciprocity Theorems of Betti and Maxwell Consider two groups of loads appli an elastic body. The loads {b 1,1} produce are the final values ofthe external forces, and if u are the corresponding displacements, then the work done during loading is rebfo-nars!fo-u Bind nbfesnavis3 fe. nas wo ord ‘As in Eqs. (2.18) and (2.20), the factor $ is a consequence of the proportionality of load and displacement during loading. Equation (6.1) may be written symbolically as 1 A=5F-u, (62) Where as in Section 2.7, F are generalized forces, and u are generalized displace ments. ‘We now consider two load systems, F and F2. While F; is applied, the work Au = $Fiom = 5 Fifa = 3 buF? (63) is done, where fix = 61 Fis the component of the displacement field tat point 1 and in the direction of F ), and 6), is a so called influence coefficient, expressing the proportionality between load and di: L. ‘When F, is added, work of two kinds >. The forces F 2 do the work 1 1 ioe An = Faw = 5 Fela = 5 6nF3 64) ‘with fon = 62aF, the component of uz at point 2 and in the direction of F, and the forces F do the work An =Fi-w = Fifio=Fibiahe 5) with fi2 = 6:2F, the component of uz at point I and in the direction of F. 104 6, Simple Beams Il Energy Principles ‘The total work done is A=An+An+An 66) work must be independent of the order in must be equal to Because of the superposi which the loads have been appl A=An+An +n 7) =Fi-m = Fifa = Fin, (68) 69) orem: When two systems of forees act on an elastic body, the work ‘on the displacements 2 is equal to the work of the forces 2 on the ty theoremisa special case of B 1. a8 the displacement at point the point k. Then Maxwell’s reciprocity theorem states, theorem, We define the ling from a unit force at bu = 55 (6.10) [Note that herein the forces as well as the displacements are used in a generalized 6.2 Theorems of Engesser and Castigliano Consider a linear elastic body subject to several loads F, (i = 1,2, points where these loads ae acting as well a their directions are Then the influence coeflicients 5, are fixed. Only the scalar magnitudes ofthe are varying. The total work done on this body is 1 , A= DDR SaR = AA GRAND no. 1 Differentiating A with respect to a specific force F, aA op = Do aFe= Sifu his (6.12) ‘im the direction of Fy. Moreover, a second differ- gives the displacement of entiation ofthis expressi respect to the force Fi, aA bron = (6.13) 63 ‘Statically Indeterminate Systems 105 describes the influence coefficients 6,,. Since A = W* for a linear elastic system, ‘we arrive at (Eq. 2.60) (6.14) Engesser’s theorem, Expressing now W = W/(f,), from Section 1.6 and the defini- tion aw = LAahi= La he (6.15) we find Wifi) _ he =F 6.16) Castigliano’s second theorem. 6.3 Statically Indeterminate Systems AA structure is statically indeterminate if the total number of forces and moments, Usually, the primary structure is selected so as to be statically determinate, but this is not always necessary. om’ /* x tx Fig. 6.1, Statically indeterminate system of degree 1: different primary structures ‘To solve statically indeterminate systems, we first may introduce the removed redundants as extemal loads X', on the primary structure, and thus get 106 6. Simple Beams Il: Energy Principles We =W°(Fi, Xe) 6.17) We also can select internal resultants as redundants, e.g. the internal bending mo- ‘ment at the central support. Then the primary structure has no restraint against bend- ing moment; tis is obtained by inserting a hinge into the beam. ah A Fig. 62 —s ‘a Primary structure 3 In ll these cases, in the real statically indeterminate structure, we have fi = 0, and thus from Eq. OW*(Fi, Xe) OX Menabrea’s theorem. ‘Again, this isa stationarity condition for the complementary energy as function m redundants X x, Since (sce Eq. 6.13) (6.18) = bx >0, (6.19) this proves ea Principle (2.57) is a minimum principle. 6.4 The Complementary Energy of Beams In Section 22, we introduced the strain and complementary energies of deformable bodies, aw = foawav, awe = [oa a. (620) v i {As our considerations ae restricted to small deformations with p © po, we find (see eg. 4.2.21) =f oi pt ut = f cudou = 35 (ouow - 5 3) 20) forthe specific complementary energy. ‘The theory of simple beams usually proceeds from the following assumptions (see Eg. 33): Oxy = Fez = oye =0 (622) “Thus the remaining stress components aeting in the cross section of a beam are: a ~ normal stresses, and r= \o2, +22. —shearstresses, 23) 64 The Complementary Energy of Beams 107 here according to different cross sections different descrip are used (refer to Section 3.2), This us to pwr = po fe s iat (6.24) ina linear theory, and the cross section cannot be disregarded. account. ares using Eq. G. Ma) Me), = ae ae Introducing tis into Eq. contribution due to the normal tess to the total complementary energy becomes a 2 7 wre fap [are aay dA (628) NM, My M, aN fea 1H raat Het fot) 27) 108 6, Simple Beams I: Energy Principles If the y and 2 axes are the centroidal principal axes of inertia, then the last three ‘terms in the foregoing integral vanish, and the complementary energy due to exten- sion and bending is (6.29) 4 (631) after integration by parts, and using Eq. (4.14). The twist 0 may be expressed by. (4.4.19) My a= FE spaafrey2)aa (632) a Thus we find i "ia | (633) In Section 3.2, we calcul shear stresses due to bending from equilibrium of the stresses at a small part cut from an element of the beam under Gen = SF rasltr2), ony =F regltn2)s (634) for plane bending of a solid cross section, and (Eq. 3.38) eee! ox =) = F3e (635) 64 The Complementary Energy of Beams 109 Calpe Remarks a (|g ©. |B-105 ° 20 T[wzo-ae] aw w3-4 o Ayeb a feeea Table 6.1. Form factor f, for thin-walled cross sections Inoducing now the form factor of shear J ing (19% 4-5 ee + Koy) dA for solid cross sections, and pti A. S00) FL HG) ac (6.36) (637) for thin-walled cross sections, we finally get from Eq, (6.25) mem aghe| Sa (6.38) ‘The form factor for shear must be evaluated for each particular shape of cross section bby means of Eqs. (6.34), and (6.35) and (6.36), respectively. Some values of typical ‘ross sections are given in Table 6.1. ‘The different contributions can be added to give Mi, MR + BE M1 Bs Gilera Gh ‘Comparing the magnitudes of the different contributions to the complementary en- ergy, we realize that whenever bending or torsion occurs ina straight member (part ‘of a structure), the influence of the normal forces and the shear forees may be ne- lected. From this, we may conclude that, for practical application, Eq. (6.39) re- duces to a -2/{ee ES Bs at z} dr, (6.40) 110 6, Simple Beams Il: Energy Principles ‘moments and torque (and thus also van- 1s8es, isthe influence of the normal forces fered. For beams with considerable curvature, we replace Eq. (6.26) by a= (i+ $)as, (641) and Ea, (627) by (642) (643) having neglected here the influence of the shear deformations. 65 Strain Energy of Beams Ina similar way as energy, we may al Introduce the speci iculated the different contributions to the complementary ‘ulate the strain energy of beams. From Eq. (2.19), we in energy (where again p ~ po) poo uf euden=G (cue + 7 : ”) (644) =m" and further introducing here the contributions from the different sources to the strains, we finally arrive at we ; f {BAul? + Edy” + Gre” ar (645) equations (see Section 3.2) - contribute to the complementary energy. 66 Application to Beams TM Pola - ===- ihe: Fig. 6.3 {alee eam st wen 6.6 Application to Beams Consider the cantilever beam ibject to a group of loads containing F ; the direction of Fis to be determined, ‘we may apply Engesser’s theorem (Eq, 6.14), and calculate aw ‘dire (6.46) ‘This procedure, however, will not work in either of the two following situations: sment of point # in any other direction than that of F ;, or ent of another point (say) k, where no load is acting. In these cases, two strategies can be used to determine the unknown displacements: 1. We introduce at point i an additional (imaginary) force F, whose direction coincides with the direction of the unknown displacement f,. We then determine 47) ‘where simultaneously forthe imaginary force Fy, ~> 0 is catied out. 2. We introduce a virtual unit oad “1” in the dreetion ofthe unknown displacement ‘fa. Then from Eq, (6.39) we get,e.g. aw: _ fv an Mam... @ ag dom ~ ewer +0n t "Gaon s™ Best for plane bending. Since, however, the resultants NV, M and Q are linear functions of Fi, their partial derivatives with respect to these Toads are then specific functions under unit loads Fe = aN _g aM 20 _5 FON Re on Maat, (649 on™ On on? re where N, A and Q are the resultants under a unit load “1” in the direction of the unknown displacement fx. We thus arive at (6.48) om [OR x 112 6. Simple Beams I: Energy Principles ‘We note that herein (since Fy —+ 0) the resultants 1V, M and Q are linear functions +n), whereas N, AT and Q are the resultants due to 13), we may further differentiate the above relation with respect to F), and Sum f {Rate Maley, Se har, 51) L where now NV, Mj and Qj are the resultants due to unit forces Fy = 1. This im- portant relation makes it possible to calculate the influence coefficients 6,1, from product integrals ofthe given kind, A brief compilation of product integrals, cover- ing the most commonly encountered functions, is given in Table 6.2. With the help of these influence coefficients, we may reformulate Eq. (6.50) to ive fe= Doki. (6.52) Furthermore, we also may use these coefficients to solve statically indeterminate systems. For a system of degree k, we introduce the k redundants as extemal forces X; on the primary structure, and thus get from Eq, (6.52) 0= bio + oui, (6.53) Where the bio desribe the deformations at point k of the primary structure under the given load system. If all redundants X; are determined from the above system of equations, we finally can describe the distributions ofthe resultants as, e.g. for the bending moment M, M=Mo+ OMX, (6.54) where Mo is the bending moment of the primary structure under the given load system. Example 6.1: The plane frame shown in the figure le hhas a fixed support and carries a ver- tical load F atthe free end. Both mem- bers of the frame have constant flexural rigidity EJ. h Determine the horizontal deflection 5), the vertical deflection 6, and the angle | of rotation y. eas 66 Application to Beams 113 finuterPae om) | ha Z t z 1 1 ok dae Stes +a) . 1 1 1 ail] Stik 4 5 ha + 2k) 1 ig ‘Dt daw dak xa Ve Loin + Bia)k peli +i) 5 ile +20) ee 2 1 1 cls Pax fae fei + ha) fin 2 2 ik Lex 2 tise: + 3ha) Tim a ai Beg age 2a 5 1 sige, on tT or oe and eRe er econ 3 Adw = wit +2" — Lot Su (1+1)40(r),] 823) where now the prime describes differentiation with respect wo radius r. Moments and shear forces simplify to 138 8, Plates and Shells Wipe = -f a+ fw+(l + nn}, ! (8.24) Mop = ~Bfv ai zw +a}. and 1 Ge = Me + 5 (Mer — My) , (625) =-n{u'+ te sasne}. 4a) Boundary conditions at straight edges Type Boundary conditions | Conditions expressed in w Clampededge | w=0 w=0 Ow Ow ao oY 0 Free edge a Pw ew _ ™Mnn =0 Oe + =0 m =0 a Ow eeu wo | Fs +0- gap =° ‘Simply supported | w = 0 w=0 edge Pw Man =0 Fe 9 oe Aw=0 ») Boundary conditions of axially symmetric loaded circular plates pe Boundary conditions | Conditions expressed in w» Clamped edge | w=0 w=0 w=0 wed Free edge Myr =0 w'+2u'=0 a =0 wr tut Sul=o Simply suppored | w =0 wa0 oe me =0 wt Lul=0 ‘Table 8.1. Boundary conditions 8.3 Thin Plates 139 Problems can often be solved by superposition of elementary cases. In general, tions of Eq. (8.23) will involve combination ofa particular solution and the solution of the homogeneous equation. The latter is known to be (see Eq, y= ate lnr+err? teyr?inr. Since the twisting moment mn. vanishes for ary conditions can be formulated straightforward again these conditions only for the sake of simp ‘mal situations. Example 8.2: Determine the maximum stresses in a ‘clamped circular plate (radius R thick- ness hi) with uniform load po. = Solution: ‘The particular integral may be taken as 1g moments at r = 0, weator +e Leratart ts Use of clamped-edge boundary conditions w = w' = Oat r = R leads to OR ogra o Bale ed (648i) 8) ooh BB and thus _ wR, 2? «= 85 Rt with moments = Re " ie Mor = 6 {ure org) 2H ial fee ge = PE {us ars) th, and a shear force q, of magnitude (see Eq. 8.25) ae Ge = — 3 Por. 1408 Plates and Shells Fig. 83 Distribution of the moments in a clamped circular plate ‘The distribution of the moments is plotted in Fig. 8.3, Maximum values will be obtained at r = R with Re Re ep = — PEE, gg =v PE, with @ maximum: etl stress (¥. Mises criterion) 3k 0m = 75g VIF Example 8.3: ‘The clamped circular plate of Example 8. at the center ofthe plate, Compute the Solution: Since there is no distributed force acting on the plate the solution can be taken from Eq, (8.26) alone w(r) =e + er? +ey7?Inr, ‘where again due to the boundedness of the deflection at r = 0 the term involving has been cancelled, The last term, however, has to be considered since here the bending moments need not be bounded at r = 0, and for the deflection we have lim r? Int = 0. mm ‘The shear force qr at r = const. i (see Eq. 8.25) a= -n{u"s tw} From the equilibrium condition in z-direction for a circular element with radius cut from the plate, and including the singular force F, we find Gp 2ar + F=0. ‘This condition leads us to pee a 83 Thin Plates 141 and thus the remaining coefficients may be determined from the boundary condi- tions w =w' =Oatr=R F = Pe, an pita). ‘We finally FRE net eee (oeaines: with moments = r me =— Ee (1404) Z}, mo=-E {vt asnng}, and shear force ug F = — Fer Example 8.4: ‘The annular plate shown in the figure is subjected to a bending moment mg at its ‘outer edge. Compute the deflection w, and the moments m yy and mo. — p= fe Solution: {Asin the preceding example, the solution can be taken from Ea, (8.26) w(r) = 09+ ene tear? +esr2Inr, with kinematical and dynamical boundary conditions: w(Ro) = w'(Ro) = 0. G(R) =0, tee (R) = m9. 1K tums out from Eq, (8.25) thatthe shear force q, at r = const. may be expressed by tena 1 a= —B{w" + tw) = —1Be*. ‘Thus, from the first dynamical condition, we immediately determine coefficient cs 1 cs a=-qger|,=0 + alr) =o. ‘The remaining coefficients are determined from the three remaining conditions ations matten yee ages co = 35g Mall 2nd) a= Bs = — ; with 2 fy r e=147401-9 Uae ats Leyte, = FP p= Fy andthe final result forthe deflection Bes a cont = FR a -P+2Imp), 8 well as moments or Bfievra-n3h, moo FB {leva 8.3.5 Elastic energy of plates ‘The principle of virtual work states (see Eq. 1.85) sw -6A=0. 827) In thin-plate theory, only the elastic energy of bending and twisting is consid- cred. From the definition of the virtual work of the internal forces, we integrate for a rectangular element over the entire plate, and obtain ow = J { = mexburze ~ myybwyy — 2Maybw gy} dA (828) 4 ‘When Eqs. (8.15) for the moments are used (isothermal case), we have AW = Bf { Aw(bu20 + 5Wyy)— { (829) an ex OW, yy + WyySW.2x — 2W,2y5W2y)} dA, and thus, we determine the strain energy w= 3B f {Aw ~ 2(1 =») (wast yy — taytt.ny) 4A. (8.30) 4 or, again using the expression for the moments, the complementary energy w= 350 2 tmi, +2(1 + v)m2, — 2vmertyy dA. (8.31) ‘The elastic energy expressions are useful in obtaining approximate solutions to plate problems. 84 Membrane Theory of Shells of Revolution 143 8.4 Membrane Theory of Shells of Revolution AA shell is an object whose material is confined to the close vicinity of a curved surface, the middle surface of the shell. To locate a wwe use curvilinear coordinates € n. In the sequel, sufficient to describe an equilibrium of the shell under: will be called membrane theory. bending and twisting moments of a thin shell must be rather small influence on the over-all picture. The membrane theory is based that they are small enough to be neglected. It meets the limits of ity when cither external moments are applied at the edge of the shell, ‘or the membrane deformations lead to discrepancies which can only be resolved by admitting the existence of small, bt finite bending and twisting moments. en loads. This description Fig. 84 Shell of revolution 84 shows a surface of revolution and which @ = const. are called parallel ci ‘curvature of the meridian, and by F, the length of a normal between the surface and the axis of revolution. Using # and ¢, we can describe the position of any point of the middle surface ‘with these parameters, eg. in cylindrical polar coordinates r(0), @, , however, e.g. for conical and cylin« stant and thus not suitable as a coordi straight generator. Due to the symmetry of the shell, we find the following principal curvatures (see Fig. 8.5). (8.32) (8.33) ‘We now considera shell whose middle surface is a surface of revolution. The inter- nal forces (membrane fore be distributed over part of the load P), ora finite force may ive when directed outward. nooRede Fig. 86 Small element of the shell ‘The equilibrium of -ment yields the equations PB Ptrree) + Te — ngp coed +1P4 =0, fae ae (834) 20) + TE + nag cosd +p =O fae Oe Wa, (8.35) ‘They are three equations for the three unknown membrane forces. Consequently, the membrane stress problem i internally statically determinate, bu it may be exter- 8.4 Membrane Theory of Shells of Revolution 145, nally indeterminate. Ifthe loads, including the reactions atthe edge, are independent of y the stress system is axisymmetric, and Eqs. (8.34) reduce to mg Ra Nyy cosd +rpe = 0, 8.36) Bane) + ngg cosd + ry = 0. Introducing here Eq, (8.35), these equations canbe solved to give nos = 5 [ (posind — py e089} Rodd, i 637) Nee = Pet Re dd. of revolution) With imation, we can calculate from Eqs. (8.37) pghR he Byes POON a ay noo(0) = hays = — Sa __ pahhe __pghR = — BEC — casa) = — pghR Lieto Neel0) = hee = OE — pghitcosd = mone (5 5 cov) ‘The equivalent stresses are calculated according to the v. Mises criterion iE 2 03 = [099 — 00090 + 73, 146 8, Plates and Shells Il, we have = const, We therefore introduce a new coordinate s, lowing transformations Rydd = ds, (Ro +00, do the foregoing formulas retain ther v I= Fa, rs)=ssina, Ryls)= ‘The loading is given as =stana Pals) = yH(1 Fone), po(s) =0. ‘We thus get from Eqs. 1 s noo(8) = havo = 2 7H stana(3 ~ 2 7 cosa) Noe(s) = hope = 7H stana(1 — ¥ cosa). 85 Exercises to Chapter 8 147 8.5 Exercises to Chapter 8 aagram. Using the Kirchhoff plate theory, ‘compute ” 1, the deflection of the middle surface, " 2. the stresses in the plate, Problem 8.2: ‘The simply supported circular plate of Problem 8. i subjected toa singular veri- cal force F at the center of the plate. Usin deflection of the middle surface and the moments. Problem 8.3: A clamped circular plate of thickness hand radi the gradient of the field in = direction. Compute the stresses that are induced by this heating \ Problem 8.4: ‘The clamped annular plate of Example 8.4 i subjected to a vertical shear force qo at its outer edge. Compute the deflection w, the shear force q, and the moments 1. and meg. Problem 8.5: strip of a plate with the dimensions given in the diagram below is builtin at one end and subjected to constant load (per unit length) q and a moment (per unit length) mat the other end. 1. Calculate the maximum displacement of the plate using the Kirchhoff plate theory. 2. Determine the point, value, and sign of maximum stress. 3. Give the equation of the deflection curve of the plate. 4. Consider an assemblage of n beams each of width b lying next to each other as ‘shown in the second diagram. Compute the deflection curve of this model and ‘compare the solution with that of a plate with the width nb. Problem 8.6: A spherical shell is simply supported at ‘The system shown in the figure is sup- posed to be a model of a soccer stadium to be constructed for the Indomitable Li- be a linear function of 9. Stability of Equilibrium. 9.1 General Remarks tend to return tits equi- surfaces of Fig. 9.1. -carrying structure which is in a state of unstable equilibrium is un- small disturbance can cause a cataclysmic change in configuration, and -minate the serviceability of the structure. turbances of a deformed configuration (bifurcation problems) or related with large disturbances (snap-through problems). 9.2 Bifurcation Problems with Finite Degrees of Freedom a spring at the other the direction of the bar axis (Fi fee soeit if r F Fig.92 PecBindeoensvantrths ecan poe Introducing now a small (admissible) disturbance f, where f < | (Fig. 9.2b), we find Ff -eft=0 on for the equilibrium ofthe disturbed configuration, and thus 92) Fa = a 03) ite disturbances f 0, the equilibrium condition requires Ff —efleosy=0, oA) jowhere | teosy = VE =F 05) From this relation, we find the general condition {F-dyi- G1 =0, 06) 9.2 Bifurcation Problems with Finite Degrees of Freedom 151 lution as before, and a second solution, on indicating an unstable equilibrium at the bifurcation point (see Fig. 9.3). | Sig. 93 Lod-efeton curve ofthe rig br. Next, we consider a system of two pin-jointed rigid bars subjected to an ax- ial compressive load F (see Fig. 9.4a). Again, the question is whether there can PRt+ a b ‘These are homogeneous equations which are satisfied by = fa = 0 correspond- {ng othe straight form, and also when the determinant ofthe coefficients vanishes Fo F-el =0 09) el F—2cyl ‘The characteristic equation FP Fle, + 2c) + crea!” =0 0.10) 152 9. Stability of Equilibrium +2at Vet+4e}, oan and more specifically (cy = 2e2 =) Fes) = 0.2981, Fei = 1.707 et. @.12) Substituting Foi, into Eqs. (9.8) find f2 = V2f; corresponding to the deflected form in Fig, 9.54, Similarly for Feua, we find fs — —/2F, comesponding to Fig. 9.5b. aaa gs Fig. 95 Deflection modes under ferent critical Festa loads Summarizing, we note that these systems have as many critical loads as degrees of freedom, that the smallest critical load isthe effective collapse load, and that the straight configuration is stable below this load. 9.3 A Snap-Through Problem AA different kind of buckling is the so-called snap-through buckling, which occurs in structures whose deformation under a load is of such a nature th of the structure decreases with increasing loading. Eventually, a point which the stiffness of the structure becomes zero, and then negative the structure is unstable under dead loading and snaps into a non-adjac shape. . Be Fes Tas ‘Snap-through problem cred. Thus, equilibrium is described in the deformed structure by F S=S=S= 755) (9.13) 94 Column Buckling 153 0.14) (9.15) such that finally F=2BA(\— cm The load deflection behaviour 9.4 Column Buckling Let the originally straight column of Fig. 9.8 be in a curved position w = w() Of indifferent equilibrium under the influence of end forces F. The classical, sim- 154 9, Stability of Equilibrium Fig. 908 Euler column in indifferent equilibrium Edw" + Fw=0, _ 0.18) «with the geometrical boundary conditions w(0) = w(t) =0 (9.19) ‘The solution of this equation, as given in elementary textbooks, leads to the Euler buckling load or critical load raat, ox Let us now apply the principle of virtual work as stated in Chapter I to solve the same problem in an alternative manner. We therefore assume thatthe shape of the curve is w = w(2), stil undetermined. The bending energy stored in the beam is has to be added to the bending energy, but when we vary the buckling deflection ‘w(2), the bending energy also varies, while the compression energy remains con- stant, Subjecting the beam to a virtual displacement dw, we see that this constant energy will not enter into the analysis. From the principle of virtual work (Eq. 1.85), ‘we thus get sw —6a= [ EJw"'bw" ae Bf =0 023) fora system in equilibrium. ‘now must express 6f in terms of w(1r) and dw. The length lg of the curved ds = Vda? + du? = dry and thus we find tom [som [Vix wFer, -t=s= f{ViFwR ipa. 7 F a (9.25) ‘The square rots approximately 94 Column Buckling 155 1 VIFF 14 Sw L wipe, 926) so that, neglecting powers higher than 2 of the small slope, a 1-3 (war, p= fw! bae 0.27) ; of the foregoing buckling problem is reduced to finding the sta- imum) ofthe potential energy I7 (9.28) We note here that the general form of the stored energy W also contains con- tributions of springs which may be added to the structure at discrete points, e.g. as supports the so-called Dirichlet boundary terms, w= Bs (u'r + Dewan) +3 ewya),| 029) { 7 7 Where ¢, and c*, respectively, are the extensional and bending stiffnesses of the springs. - 9.28) can now be used to determine approximate so- approximate function which for a differential tinuously differentiable and which satisfies at 1e geometrical boundary conditions. We will call this an admissible function, IL is observed then thatthe principle is equivalent to finding, among all admissible fubctions wi, those which make the quotient (9.30) stationary. This is Rayleigh’s method for computing an approximation to the lowest buckli ‘The approximation obtained by this method is never less than the true sprove this method by introducing admissible functions, e.g. 031) ‘which are still functions of r undetermined parameters @,, whose relative Values are to be selected, in conjunction with Rwi(1;aj)], such as to make IT stationary. The necessary and sufficient conditions for 617 to vanish are OF M9, G@=1.2..07), 02) 156 9, Stability of Equilibrium representing r linear homogeneous equations inthe a {L-FMja=0, 033) wherein bay = f Bade + ct (zialoo)+eehleryik) 4 0 (9.34) Ma= f wihae. ‘These equations have solutions other than the trivial one only ifthe determinant of the coefficients of the a; vanishes det(L — FM) =0, (9.35) yielding a sequence of r approximate buckling loads. Each of these approximate Critical loads is never less than the corresponding one of the first r true critical loads. Ths is the so-called Rayleigh-Ritz method for computing approximations to the r lowest eigenvalues. Example 9.1: Determine the critical load of the Euler other end is simply supported and under a compressive load F. 1, The simplest admissible Function i() fulfilling these conditions would be W(2) =e*(2-1), ‘here cis a constant coefficient which will play no role in the solution, To substitute this shape into Eq. (9.30), we need ; = (3? — 221), a" (2) = (62-21), and. fesenecaswese, fterte- 220 y 94 Column Buckling 157, ‘Thus, we find as approximation for the buckling load Fea < Ri] =0 8. siderably greater than the correct value of 20.19 BJ/l2. 2. A better result is obtained by using the shape Peers: (e) = ex*(2? ~ F214 37), ‘which in addition to satisfying the three geometrical conltons above also satisfies the dynamical condition M()=-Es ‘A function satisfying all boundary conditions will be called a test function, We further note that test functions are obtained e.. from any deflection curve of the system under consideration, since they satisfy all boundary conditions. Here, we took the deflection curve of the beam subject to a constant distributed lateral load 4p. Introducing this function into the Rayleigh-quotient Eq (9.30), we arive at BL Fe $ Ri] = 215 3. The function lz) = e(cosg ~ cos), § = x2/21, iis an especially good representation of the buckling mode, leading to the result Fen Rid] = 20235. ‘Ts exceeds the correct value by only 0.2. 4, To obtain a Rayleigh-Ritz solution, we introduce an approximation W(x) = yw; (x) + ey ) of two linearly independent admissible shape functions , w(2)=2%e—1, = 429 — 3241, = 122? — 6rl. From these, we find 1 ta = [Ba )? dr = 4E Pt Lu= ih EJ wiwl dx = ABI ba= f Bs(u)de= Fest 3 158 9. Stability of Equilibrium and ‘ M = ['esiitae= 2 dhe 2 F 1 eee [ BJ wyws de ar 7 tee ice ckan a Introducing these values into Eq. (9.35), gives has the two solutions 16,1 daa 545 Vi, and thus A = 2092 BJ/l fy =107.1 Bd/l 9.5 Exercises to Chapter 9 Problem 9.1: 1. Using the quotient 1 - f Bs de ee Fei < Ria] = T fitar with the admissible function iD = e&(1~€), €=/l, ‘9.5 Exercises to Chapter9 = 159 2. and the energy method, ic. the Rayleigh quotient Jes? ae Foi $ Ria] = with the same function = e€(1 ~ &), ‘determine approximate solutions forthe critical load Fx. for a column that is simply ‘supported at one endl and pinned atthe other (Euler column, second case). Compare these results with the exact solution, F P= KBD é ee 1. using the Rayleigh quotient with the admissible funetion w = a €? and setting ¢ — 0 and ¢ —+ 00, compare the results obtained with the corresponding exact solutions. Problem 9.3: Far the system shown in the figure, deter- mine om the Raleigh quotient approx: “i -——————_, =F imate solutions for the critical load Foy Problem 9.4: For a column that is fixed at one end, 3 pinned at the other end and loaded by a Je=e= === 2 = a ‘constant normal force per unit length of in- tensity n as shown in the diagram, com- pute eel) es 1. the Rayleigh quotient for the normal- force n, 1609. Stability of Equilibrium 2, From this quotient, calculate using the function i = a€2(1—€), = 2/l,an 10. Some Basic Concepts of Dynamics approximate solution for the critical load 3. Determine a polynomial of degree 4 and use this polynomial with the proximate solution for the critic fies all the boundary conditions lotent from. to compute an ap- 10.1 Principle of Virtual Work In the following last chapters, we will again widen the scope of our investigations by considering the motion of particles, masses and bodies, and the forces acting on these moving bodies. In Chapter 1, we discussed the equations of motion and the balance of energy of bodies in motion (Sections 1.5 and 1.6), and we introduced the principle of virtual work for statical problems ( 1.7), We shall now extend this prineiple from statics to dyn adding to the equations of motion - on the right-hand side - the acceleration (inertia) terms, which have been disregarded when determining Eq, (1.85). Again, starting by multiplying the equation of motion (Eq. 1.63) by Su, we arrive at + ¥ ‘The right-hand side now gives : D D> forsuav = 2 foniduav ~ [ov 2 (ou)av. 02) v + ‘ Since 6(Du,) = D(6u), the second term on the right side can be written as the variation of the kinetic energy [or 2 u)av = fonsmav = 6 [Eovzav = 58. (103) ‘ ‘ ¢ ‘Thus, with the result of Section 1.7 in mind, we find Ad Ad ag arden (104) the of virtual work for dynamical problems. We note that this principle, dependent of any constitutive relation, 162 10, Some Basie Concepts of Dynamics 10.2 Hamilton’s Principle tegrate Eq. (10.4) with respect to the time t between two definite ‘ta, which may be chosen arbitrary. The integration of the last this case, the boundary term drops out, and we obtain Hamilton’s principle [Caw + 5a+68)a=0 (105) Introducing now an ‘material, where the stresses are derivable from a strain energy potential w(: id assuming moreover that the forces s4 and by are also derivable from a potential, we can introduce a single scalar function M=W+%= | pwlew)dV + neva + [eadA, (10.6) ‘ ‘ A where ae ee oF, EA. (107) A system of this kind is called a conservative system with 6 = —6A. Since we have 611 = 6W -6A, (10.8) Hamilton’s principle, for the case of conservative forces, may be written as follows. sf (e-ma=5[ Lao, (109) where L = E IT the Lagrangian function. ‘This principle is very similar in its character to the principles discussed so far ‘Once more, we have a stationary principle, where now the quantity which has to be ‘made stationary is the time integral of the Lagrangian function L. 10.3 The Euler-Lagrange Equations See oe ee a eee (6W"= 0), and ofthe q, such tha they actin the direction of the qi, and 103 The Euler-Lagrange Equations 163 54 = Q.5a, (10.10) is the virtual work done by these forces in the direction of the virtual displacements ba ‘The essential characteristic ofa system is that the objects are coupled, and hence restricted or constrained in their motion. If we consider, e. . an ensemble of n rigid bodies, we have the degree of freedom A=6n-r, (io. Where r are the number of constraints. These constraints may have the form (10.12) straints are moving. If the constraints are fixed, ‘We note in passing that constraints which are differential conditions are called non-holonomic. In the sequel, we ‘consideration to holonomic systems. Finding the minimum of the de (10.14) D Sai = | Oa), (10.15) the second term on the right-hand side of tis elation can be transformed such that instead of Eq. J, we have (555) ~ a 3 (sp) om (10.16) to Eq. (10.9) yields sf Bae (Mane toc ‘The first term on the right side drops out, since all variations are supposed to vanish at the two end points of integration. The remaining term has to vanish, too, and thus since the 6g; are independent, this leads to 4 simultaneous equations 164 10. Some Basic Concepts of Dynamics dt \ 0G: Ou ny ITeseh gon Ele egtoa eo Oe collet yee Since the potential energy # = (q,;¢) is no function of generalized veloci- ew tC pre 016) Pu OS Ln a a eae sa on 0.18) D (dE OE OP BH) dea 2% (F=1,2,...,2) 0.19) Fornon-conservative forces, we start from Eq. (10.5),and then using Eq, (10.10), ‘we realize thatthe second part of Eq. (10.19) is also valid if the @ ; are not derivable + @=1,2)...,2), (10.20) ‘We finally note that Eqs. (10.18) and (10.20) are also valid for systems of de- formable bodies. 104 The Lagrangian Multiplier Method Given the Lagrangian function L expressed as function of A+ r time, and given r holonomic auxiliary conditions of the kind ( ‘times tedious to eliminate the In this case, following a suggestion of Lagrange, we simply modify L by adding the auxiliary conditions L=L+ fey (R=1,2--57) 0.21) where the Lagrange multipliers \y are r additional variables. Introducing now this ‘modified Lagrangian into the variational principle gives the following + r equa- tions of motion D/OE\ OF _ fe a (am) ~ ay Oe (10.22) ‘unrestricted variational principle. | 11. Oscillators With One Degree of Freedom 11.1 Undamped Free Vibration The simplest oscillatory system is one of a single degree of freedom, the motion of which can be described by a single coordinate q. A mass 1m suspended from a spring, of negligible mass represents such a system. t Fig. Fin eau wees le force cqy, where cis the spring stiffness, defined as the force required per unit exten- Impression, and q.; is the statical deflection of the spring due to the weight ‘evident then that by choosing the origin of the coordinate q atthe state um position, only forces due to displacement from this position should be considered. From Newton's second law, or alternatively using d’Alember’s princi- ple, we find mi+eq=0, at and G+upa=0, ‘having introduced the circular (or eigen-) frequency Beale «a.3) ‘The homogeneous second-order differential equation has the general solution a(t) = cosinwot +c cosunt , describing a motion with the period 2 EE r= 2am /2, ais) Ps ¢ ‘and frequency ws pote f=T=7 5. 16) Equation (11.6) may be expressed in terms of the statical deflection gx by noting that equ = mg, from which we obtain a Rye f= Fora system of mass m,the mass moments of inertia are defined analogously to Eq, (3.22) asthe integrals = [tern Oar Bay Boe where the 2 are centroidal axes and the integrals ae taken over the whole mass or volume, respectively, of the system. ‘We note that duc to their definition the mass moments of inertia 8 ~just as the moments of inertia JJ;x - can be interpreted as components of a second-rank tensor, ato) a =— Of the system is determined from the influence coeffi- fer to Section 6.6). The deflection qj at a point i of a a2) Moreover, in cases where the system contains elastic parts - like beams etc. - itis also advantageous to determine the differential equations of the system using ‘methods from statics. We will demonstrate this with a simple cantilever beam with ‘one concentrated mass at its end. Starting with equation nitude of X to describe the inertia ), we know from d°Alembert’s principle the mag~ X, = mii ‘Thus, we find ania) ) mii + eqs =0, the differential equation of free vibration of the mass m at point i. ‘This method can also be applied to systems of several DOF. From statics, we know th = 8X1 + b2Xa +--+ binXn 2 = 8X1 + bxaXa +--+ banXn dn = Su X1 + bnaXo +°+°+ Sn Xn fora system of n DOF (refer to Eq. 6.52), Le. a= 6X. 168 11, Oscillators With One Degree of Freedom Herein 6 is the maltix of the influence coefficients, and according to d’Alemben’s principle X is the vector of the inertia forees: ~m i, and ~Oydis respectively ori matrix notation X=-Ma. an.) Introducing this into Eq. (10.16), we finally arrive a= -6Ma. ais) Multiplying this relation from the left-hand side with 5" = C gives, Ma +Cq=0, (11.20) the system of differential equations describing free vibrations. system shown in the figure. The spring has a stiffness of c= aFJ/I*. biases | 2 fee Ae ey 12) and determine the influence coefficient 6; of the system for ‘acting vertically at the position of the concentrated mass m. The ‘moment diagram turns out tobe (refer to Section 6.6) le t fa iors tate Sikes 12 Bdé =E EE (+2), ag — 8EI_a oe 0 = nl a+ 12° 7 7 uals with bounds 0 < wif < “S22 forthe fee, and the simply supported beam, respee- tively, Taking a = 12 (rele to Example 62), we et a _ MES w= mie 11.2 Damped Free Vibration = 169. 11.2 Damped Free Vibration Energy di the dissipation and the one w' the velocity. a a Fig. 1.3 Fa Damped system of one degree of freedom takes place in all vibrating systems, and the force associ ied damping force. There are different types of dampi the simplest to consider is a viscous damping propor ‘The equation of motion for the damped free vibration is mij+dj-+eq=0, ay or G+ 2Duog +u8q=0, (1.22) where D= sé (11.23) is the (dimensionless) damping factor. Equation (11.22) is a homogeneous second-order differential equation which ‘eqn be solved by assuming a (complex) function ofthe form at) = ae" (a.24y ‘Upon substitution, we find that s must satisfy the characteristic equation F(s) = 6? +2Duys +45 =0, (11.25) Which has the two roots, 12 = u9(- D+ VD?=1) (11.26) Hence, the general solution becomes a =mer bere, az where co and ¢; must be evaluated from initial conditions. ‘The free motion of the damped sys nds on the numerical value of the radical of Ea. critical damping as the ‘The actual damping of the system can Case 1, D > 1; The radical in this case is real, and is always less then 1), so that 170 ‘Oscillators With One Degree of Freedom ‘51 and s2 are negative. The displacement q then becomes the sum of two decaying, exponentials (Eq, 11.27), with — fo + qu oe DE ain (11.28) oy = — fot aot ( ws = t ov DF = ‘Such a motion is non-« eae led aperiodic. Case. <1: The radical in ‘imaginary, and s can be written as (11.29) where w=uovi—D? is the circular frequency of the damped system. The general solution then a) = € P" (gainwt +e co8wt), ‘and the motion is oscillatory with decreasing amplitude, with Gy + Daowo = (1.32) cease (critical damping) represents a transition between osci oscillatory conditions. The roots $1 and s2 approach each other, and to ~wo, The general solution then has the form (go(1 + wot) + dot]. 1.33) to the aperiodlic motion of case 1, however, it has the smallest and hence returns tothe equilibrium position «an.34) lation be described by Eq. (11.31) the logarithmic decre- ) can be determined in terms of the damping factor a = Duo 1.35) Since the period of a damped oscillation is equal to Qn On r= i w ~ wovI= Dt (11.36) 11.3 Forced Vibration with Harmonic Excitation 17) the logarithmic decrement in terms of D becomes D vi-D’ ‘which can be approximated for small damping factors as oz nD. (11.38) ‘This allows the determination of D from a number of cycles of a simple oscillation dan (137) finally mention that any harmonic oscillation can also be described through a(t) = 4 00s (wt +) ‘where now is some phase: we find w= Goose, dof = ~Gsing (11.40) ¢=ye+3, ge arctan Se 11.3 Forced Vibration with Harmonic Excitation and @ isthe amplitude, Compared with sly damped spring-mass system (see Fi Qeos 12. The differential equation of gradually disappear because of damping. The steady-state oscillation be described by the particular solution a(t) = 4 cos( Mt + 9) (11.42) 172 11, Oscillators With One Degree of Freedom where 1) = 2/a. For convenience of presentation and discussion, this solution is, generally reduced to non-dimensional form. We therefore introduce the magnifica- tion factor of amplitudes sa SN cass PFD ae taln) = 9. (1.45) Fig. 4 Magnification factor of ampli- todes = oes Ws is ae as Fig. 1.5 10 zo 17 Phase responsé The maximum of V, is 1 Vena = = Vina apy t= VID 146) for damping factors of 0 < D < } V3. 11.3 Forced Vibration with Harmonic Excitation 173 A vibratory system is sometimes excited by a prescribed motion of some point ‘u(t) be the harmonic motion of the support point of the ta L fu(t) Fig. 1.6 System excited by base motion the differential equation becomes mi +d(q— i) +o(g—u) =0, a7) ‘which may be rearranged to G+ 2Duy G+ hq = uh u+ Wuyi. (11.48) We will now the method of complex algebra, which often simplifies the (aso) asp (as: the magnification factor ‘To find the phase angl ef? =cosp +i c ‘and again find from _ Im) lane = er cass) 174 11. Oscillators With One Degree of Freedom 11.4 Vibration Isolation Vibratory forces generated by machines and engines are often unavoidable; how- ever, they can be reduced substantially by properly designed springs, which we refer to.as isolators. Let the force @ acting on the system of Fig. 11.3 be the excitation source to be isolated by the spring ¢, The transmitted force through the spring and the damper is Qr= cGy + 4DBP. (ass) tude q developed under the force Q(t) = Qcos.2t is given by Eq. bove equation reduces to = QV1+4D%? Van) Since the (1.43):, necessary for 2 to pass through the point of resonance. Ivher/@ fx sea Fig. 117 Obs Ta 1s 20 Rs Force transmissibility 115, Exeeciesto Chapter 1) = 175, 11.5 Exercises to Chapter 11 Problem 11.1: ‘The system shown in the figure consists of rigid beam, a spring, a pulley, a mass- less string and two point masses. The point isnotattached tothe beam.In | % shown, the spring is relaxed. 1. determine the ratio amplitude of the - vertical vibration of the vehicle to amplitude of the road 19 (max /to) a5 function of the horizontal velocity factor D such thatthe ratio zax/ up does not exceed 1.5. Here, wo denotes the natural circular frequency of the system. 11, Oscillators With One Degree of Freedom Problem A rotor of an ‘mass 7n; = 300 kg, and eccentricity of the center of mass ¢ = the transmitted force does not ex the amplitude of the oscillation for this case. Problem 11.4: ‘The vibratory system shown in the fig- ure is excited by a prescribed motion at ‘one end of the damper (with the property & = Ame), Determine the different mag- nification factors of the system. ine with total mass mm = 1,000 kg has the following properties: 0.1 mm, speed is n = 1,500 rpm. Determine the stiffness ofthe elastic foundation such that 20 % of the exciting force. Moreover, compute 12. Systems of Several Degrees of Freedom 12.1 A Typical Example ‘There are two methods available for obtaining the equations of motion of a system, Figure 12.1 shows by) gi but restrais ‘ma, Which moves 178 12, Systems of Several Degrees of Freedom = mudi + mal + de)» a OE Ba. = mah + aa), 3 ni aa 4=1,2, OP Fem ei— Fosin xe ne mig + molds +) + erg = Fosin Qt (125) +) +22 = 0. a Itis easily seen that Eqs, (12.5) and (12.1) ae linear combinations of each other. 12.2 General Equations and Solution ‘To find the frequencies and modes of small oscillation of a system with n degrees of freedom (DOF) about a configuration of stable equilibrium, it is not necessary to consider any damping. The equations will always appear as an array of n linear equations of second order Ma+Cq=0 26) for free vibration, and MG+Cq= F(t) 2.7) where M = MT is asymmetric mass matrix. For conser (12.8) Juency wp and the phase angle ip are the same for all qs. Sub- stem of homogeneous equations. ‘This determinant, which is called the Lagrangian determinant, may then be ex- panded, and yields a polynomial of nth degree in «3, called the frequency equation. Cs CR ea a Se EP ‘The n roots are the n values of wo, the n eigenvalues ofthe system. They are usually arranged in order of ascending magnitude, the lowest being called the first natural Equation (12.10) has been derived from a system of n equation: there remain tution of each value of «2. For each natural frequency the different values of the corresponding to that frequency describe the mode of vibration. Thus with the st eigenfrequency, the system oscillates in its first mode; and so forth. 12.3 Forced Vibration with Harmonic Excitation Let us consider the forced vibration of atypical undamped two DOF system such as shown in Fig. 12.1. The equations of motion of this system are (see Eqs. 12. erg — cag = Fo sin 2t +i) +em =0. As with a system having a single DOF, circular frequency agreeing with the exci gq =Gisin iM, 2 = dasin Ot. 2.12) Introducing these into the above equations, we find a2tny steady-state forced vibration will have a -ah=Fo 12.13) ’ +(—ma? + ¢2) da = 0. 3 the two unknown amplitudes (1, da: with the solutions 2)/D y 12.14) dy = Fom2/D, where D is the Lagrangian determinant of the free system, that is a-me ce D= (2.15) =m? — mF natural frequencies; and, at these two frequencies, both d and dy become Infinit, ie. there are two resonances (see Fig. 12.2) However, from Eq, (12.14), we also realize that the amplitude may vanish for 2 = ma? ‘Thus the response of the principal mass 1m may be reduced to zero, so that is does ‘not move at all, This describes the behaviour of a dynamic absorber for the forced 180 12. Systems of Several Degrees of Freedom 12.3 Forced Vibration with Harmonic Excitation = 181. a Th ini eneryof the system i = Pitt omet = 2 B= Smt + mat = hm 1 amg? -+ La [dn + Lae conan? + (Lénsin en?) =pmdi+5 = hd? f+ La + 2Lauin cosan] ‘Thus we find (for small values of qa, ie. 42 <1) vibration of mass my under harmonic excitation, and with a frequency £2 close to | rine fa LA ya oe the resonance of the system c), m1. Adding a second system ca, 7g to this system gene eee close to resonance allows to determine e2 and rz such tat the amplitude y may ‘thn guid Ath elec gl ire Wk ti fen vanish. inthis system under an admissible virtual displacement, since Example 12.1: $A=¥Qeda ‘The horizontal oscillation of a tower may 4 fe dea by as emcan peig- Tie defines these generalized forces (see Eq, 10.10) ‘mass-system of one DOF. Due to some har- “ * Faces 2t monic excitation with the frequency £2, this yma system undergoes a forced vibration close to the resonance of the system. To improve pee oo the situation, a pendulum as dynamic ab- — ? sorber has to be added to the system. Deter- at ‘mine the dimensions ofthe pendulum. Solution: ' Ayia sine From Eq, (10.19), describing the motion of the system are the Lagrange equations fait Foeos 2 Fig. 124 System with forces (left) and virtual displacements (right) 6A = (Fo cos 9t — ons — bin) 61 — magLae 542 = Qi dan + Q2 S02 i 7 ‘We may calculate the following vectorial quantities OE _ ‘ eb__w eae Sma ma Introducing this into the Lagrange equations, we find tt - cos a2) 1 a (im, +2) mal bo’ 0 Fycos 00) Fig. 123. System with velocities a+ at a= mL maL?, 00, 0 magb, o D (aE) _ oF We first determine the eigenfrequencies and the modes of vibration for an un- Hea Dalen a eal damped system. 18212, Systems of Several Degrees of Freedom From Eq, ( e—wp(mm +m) —umal, 10), we conclude =0 swims magh ~ wma? withthe frequency equation @ Lalo? ta (14%) | 4 tee = wi-od ft vat (14 22)] rated, where 2? = c/m, and 23 = g/L, respectively, are the squares of the eigenfrequen- cies ofthe two single DOF systems: initial spring-mass-system and pendulum. ‘The solutions ofthe frequency equation are (€ = mm./m1) a} — (1 ~ §) a303, iam flea teal] at late ter ‘ad mareorer 2-40" p+e4 VETO] for the special case of identical eigenfrequencies of the underlying initial systems, ie, o=a}=0%. Iewill be shown that tis special case is of great importance for the behaviour of ‘our system. Fig. 125 Solution of eigenfrequencies vs mass rela- o 1 2 3 4& € ting ‘We now design the dynamic absorber. For the forced vibration of the (undamp- ced) system, we find ee ee, -Pmb mg mi?) * \ o From the second equation, we determine gL, Ph +0-Pe=0 — @= 8 124 Exercises to Chapter 12183 tts ee a uation Ee aft [e— (my +ma)] LAE yt — mada = Qo, sd inaly t= = Van), wes am. yells eat a = 818 tm) = 2 Vien). ‘tturms out that there exists a solution to our problem with 2 4 =jeal — 2a s+ IL ms Tart L [eae The amplitude ofthe motion ofthe pendulum is then given by at % Qo A L 1 ee =i) ae LM hy clots bab = 2 Valnsm= 1) =—2 oy = -Qo = Qo To geta small amplitude, we choose a large mass m9. 12.4 Exercises to Chapter 12 Problem 12.1: Compute‘the natural massless frame showr 8 concentrated mas with constant xu 1 4 Problem 12.2: “wo identical oxilators each with mass : mand length) are comeced by an elas- of ] tie spring a shown inthe igure. Co ‘the natural frequencies for small tions. 184 12, Systems of Several Degrees of Freedom Problem 12.3: For the system shown in the figure with #)_@ constant flexural rigidity EJ 1. det ising the influence coeffi- \ is the differential equation Re illations ofthe system. 2, Establish the eigenvalue problem, 3. and determine the natural frequen- 1 L cies of the system. Problem 12.4: Solve Problem 12.3 using the Euler-Lagrange equations for conservative systems. Problem 12.5: AAs shown in the figure, a rigid disk of diameter d, mass m, and mass moment of inertia 6, is fixed to a massless flexible shaft of constant flexural rigidity. Compute the natural frequencies, and the different modes of vibration. t=d=1m, G=7.85kN, BJ =400kNm? Problem 12.6: For the system depicted in the figure, the ‘Masses 7), m2 and mg, the length / of the strings connecting ry with ma and the tion q3 = 0, the spring i ing and friction forces are assumed to be negligible. Determine ‘nian function the differential equations describing the motion of the system if the motion of ris given by 41 = fat 13. Answers to the Exercises Chapter 1: Basic Concepts of Continuum Mechanics Problem 1.2: ‘The eigenvalues are Ga: 01 = 30, 02 = -6, 0 = 12 Gy: 01 =25, 02 =20, 03 =10 Ge: 01 =40, 2 =40, 03 =0 fei ey=—l0-4, ep = 10-4, eg = 410-4 61 =3-10-4, ep =2-1074, eg = 10-4 fet e=10-4, ep = 10-4, ey = -2-10-4 Problem 1.3: 8) 91 =178.06, oy = ~78.06, gy» = 19.3° b) on =130, ayy =~30, a2 = ~100 ©) Toax = 128.06, 1 = 6439", 22 = oyy = 50, afi ® _ 1/00 Selo ety) 3. yo = 28.15, e1lo = -1- VIB, alo =~1+4 VIS Problem 1.5: 1 fas = Ear Ew = 5 (280+ 2 6)y Ey = ch-+e2 — Regen — Renee ~ Besta) 2 eet y= Zotete) B= 5 1 saa Gleotentedt 3 Veh +e tet cae e180 Fata 186 13, Answers tothe Exercises Chapter 2: Elastic Material Problem 2.1: OY areas 1. om = 50+ 5 VOR+ ar 2. Oy = VOrF ATE 3. og = Verran)? 4. oq = VFS) Problem 2.2: Ba W = 54 6+5v2) Problem 2.3: He Fa(4+2V2) 5 Fa(8 +62) aa HA eatahe aie Problem 2.4: bu =4aboa, by =0 Problem 2.5: 6v = 00a, 64 =0 Problem 2.6: 6 = 22HVDFa 5, _2Fa el Ferg SO RA, Problem 2.7: Fa Pa $= ypq (V2 Ds Sy = 5p 8-¥9) Chapter 3: The Theory of Simple Beams I Problem 3.1: - oer mar aera kek ) wo H0, Aad = 50, =I = ed 5 Sy by eel b) vo= 45, = 5a, a= Gard a ot ae 2 o7(0,0,0) = (20+ 2V08) & 3. ra( 0,0, 4) = (40+ 5 VRB) E 4 0m(040,5) = 05 Problem 34: p= 26.45° o> tain rnin = £51.41 Nim? va Problem 3.5: eee ee mm = 98 ba’ OBB Sa Problem 3.6: 8, 5 2 Mes tae = Bigs fm =O T= FT 3Q ra 2) tam = Bag le 18813, Answers to the Exercises Problem 3.7: _ 3Q Ni +My | _ BUS 9) Tm = hb, hb, + 0662’ © Ti, + 6082 3Q. a eno >) tan = 57 6) fea epee rms 805 “28 tm = 2H). 200? +a? + nar) ) Ton = F5(later)’ ©" datar Problem 3.8: 2) Bhin(in)= Met BJuy(22) = MH (1-2 - 2) _ alte, tty _ Mon, om, at ») Bsa) = SF (4- FB) (sot +H), _ ott a2 428) Mol (,_ 2 Eos(an) = 25 (52-557 +338) - “FE (1-2) Problem 3.9: _ 3EJAaby OF ST+ AR BJes(an) = f cotter — 3), BJalaa) = ~ Zealoa — Daa +21) 5 Ey Bho(e) = £ Fa (5-23 a U Esrws(ea) = & FP(5- 2-0 + Problem 3.12: w@) = fle-0? 13, Answers to the Exercises 189 Chapter 4: Torsion of Prismatic Bars ‘With a linearly varying stress across the thickness of the profile. Problem 4.3: ‘As sum of the shear stresses due to the shear force and the additional torque, we tions ofthe shear stresses are: 1, due to the shear force Q 2. total shear stress at b-b 1.5MPa Q \ 39 t en g ETT 3g Bal hv 19013. Answers tothe Exercises 4) [Flaws = 18.86 N/mm?, pq = 6.21 -10-° =17.64Nimm?, gq = 4.35- 10-8 = 74.07 Nimm?, ga = 28.16-10-% 4) |r|max = 60.06 Némm?, ya = 21.21: 10% Chapter 5: Curved Beams Problem 5.1: A RS FR a) w(y) =crcosy + easing FAG yey, 4 =0, = FEF PR ug) = Z (psiny-2+ 2008) FR [yx rh » w(e) = $E [(F -1- 2) sing +ony © (nai -1+ £ (rxing-20m9)], PR [ox 3 Whe) = Fs [(J -1) ep Fane ,@ H1- 3+ (rsine+200)] Problem 5.2: : " MR MR 9) uP) =asinpteacwe+ LE, 4-0, a=-2E, = ME (p—siny) u(y) = - 2 (e-sing FR by we) =— Fp esing, FR uly) = apy (ing — pe0sy) ‘We note that herein (in the solutions for problems 5.1b and 5.2a) for beams of small curvature we have neglected terms J/AR? compared with one. 13, Answers tothe Exercises 191 Chapter 6: Simple Beams II: Energy Principles Problem 6.1: Pa We=@+Vv9 54, iv= Problem 6.2: vant Problem 63: 137ql* 1844 ov = aps Pgs Problem 6.4: ‘The distribution of the bending moments is Problem 6.5: ‘The distribution of the bending moments is. OF F a a Sat, BHF, B= S= = -6F =F F aE So = = =F, =F Sn =0 Su=0, Su=-F, Su=F Su=-F, Si=0 19213, Answers to the Exercises Problem 6.7: 3EJo® “Psa Problem 6.8: ‘We introduce the two redundants X's, X and compute 4P F X= Fy (5+ v2), X2=- Pica - 432) N= i" 038Ft 02087 « M % aN Problem 6.9: i Bee en\2” 4) BF Problem 6.10: i emilee Pet a) BT F w MOT, 2. dee= Gr Cas = pens O12 5 or Problem 6.11: (1+ V10), y= -35.78° 13, Answers tothe Exercises 193 Chapter 7: Two-dimensional Problems Problem 7.1: =hy ) Pay a fo [Phas b) ro B [ve ge Problem 7.2: Lee a) F=— 5000" @8°(pi — Po) (2p = rob?) b) Pealarterrt, 6 = GAR, 2 = BRE, Po=—20, n=0 Problem 7.3: P(r) =a, Ine + 037? + agr?Inr oy = BF (evintja)) ag =~ [yt —a2 +2(6%Ind— a? na)] Ue GIs ty = ts =0 Problem 7.6: Q 3Q 4y? aia iedee cheese 19413. Answers tothe Exercises Problem 7.7: 2Q Orr = — FF, SING, Fy =0, Arp =O Problem 7.8: Corre = 300, 9= 1/2, r=R Problem 7.9: as EsEp 1 Epa + Ad/2)(1 — vs)(6? — a?) + Es|(1 + vp)ab? + (1 vp) Ad > = 104. 1. @> SS =10417K t AdEa _e@ 2 Om = Ta) +s) (: 9) AdEa e ee = Fav) + PUSH) (+ z): corm er(a) = ~112.90 MPa, o'pp(a) = V41.13 MPa 3. Ors = Goes =~ (Ua? : aren = Hath —#/r°)K, . vey 4 2(1 + 8%/r?)Ky Orep(@) = Ores(a) = 9746 MPA, op p(a) = 121.82 MPa 4, Take the solution of part c) and rep! tet 4 T=a5 . Orrp(a) = Arrs(a) = ~101.01 MPa, eo (a) = 126.26 MPa 13, Answers tothe Exercises 195 Chapter 8: Plates and Shells Problem 8.1: _ oR [5+y (840) rot y wie) = a [He 2a Bt 2 ontna) = | Baty) (1- Fe)» 2 —. Goolts2) = 3 PR a+sy [Pee - Be Problem 8.2: FR [3+ vl pr wt) = es [HE (- B) +E S| male) = 04+) E nk, F r mop(r) =— = [(1+y)inF -1 +44 Problem 8.3: 8) Constant temperature field dp ult) =0, oer = opp =-— Bato, I-v by linearly distributed temperature field 0 = W(t) =0, Mee = mye = —B(L + v)a0, The stresses will be superimposed to give: oor = Bop = = EH (dy +042). Problem 8.4: w(r) = — BE ri (47mg + (1 p2\(0-+29) + 207" np} seit ar) ==, mer) = BE L(L+- oF inp + (1 UAB) + (1+ VOIP}, molt) = BEL (1+ Pimp — (1 W)CL + von-%) + (1+ yy@Inp}. a 2. 2=0, losalnn = 53 (MR+4R0), Oy = Vee, Ory =0 3. wK(@)= a [3(mp + 4gna)2? ~ gaz) 1 4, wP(e) = Ga (Mn + Qna)z” ~ Qn2"), wk ge = 0889 fory=0.3, nbgr=nQr, nbmg = nMp and beams with a rectangular cross section, Problem 8.6: Po =0, Pn =7R(cosd — cos Io), 1-cost# 1 now ae [Tar 3a 1 1 cos? Nigg = 1? [eos — } cos:dy - +206 # hia [ 2 Bain? 7 Problem 8.7: i = sa Uinta — hada + (ha — hn) ae = Matec ep andcael 4 00 = — Gaye lata — haba) (co8 0s — cos 8) +(ha — hi) (sind — sin dy — 0 cos 0 + V1 cos¥)) = pg ~ vs d +(ho — hy) [sind — sind; — 9 cos (1 + sin? #) +9, cos] Nee {(hi#2 — has) [008 0; — cos #(1 + sin? #)] For hy = ho = h, 1 = 0, we find pghR sin? 1 Moe = pai oon — =] noo = PA (eos — 1] 13, Answers to the Exercises Chapter 9: Stability of Equilibrium = 10/x? =12/x? Af [p04 0 Vine ote rm] ‘crit “9 Problem 9.3: = EJ emt L R=stn) Fe, x= 2 Fa 2 [iss +1) - viv 7660 amr aoe] BE fEsw"ae 2 EJ ~2/), b=-3a, R= 3675 197 198 13, Answers to the Exercises Chapter 11: Systems of One Degree of Freedom Problem 1.1: oe) 9m’ ot) = $1 eostut) + gai K K r= Gr Ma—a = 2 lt) = wo costwt + Pam), 1. G+u%=K, w= ‘The force re to the foundation is Q(t) = g(t) Vn) = Sa = =02 + n=v6 + w= 64.145"! naailsaven! 9 =0.036 mm, gg = 2.39 mm Problem 11. Chapter 12: Systems of Several Degrees of Freedom Problem 12.1: EI ET oy = 0807 1) 5, wn = 2.824) 13, Answers tothe Exercises 199 Problem 12.2: wid, ody 2m ne ees eae Problem 12.3: n+ 4m +n =0 El $2 +6\n + 4am = 0-55 abaya = 2. (40- et) +an=0 5 6g + (4a- 3x) e =0 3 ooaf Ses vamp} ‘The system is of 2 DOF. Following 9), we find with 2 1 a 4p. cs ate Jb = =t Eléu = 58, Edéa=EJb=-50, Edda = 5. ‘The mass moment of inertia is, 2 om Femi. ‘Thus, from the Lagrangian determinant (Eq. 12.10) ‘The modes are determined from the ratio G2/¢1 1-dume? _ { -0,514 520? 31,02 200 13. Answers to the Exercises Problem 12.6: Index (omy +a +s) + drdals cos 42 + drdsms eee! +7 BP mat 5 = magl(1 — cos) + joa L L=e-a 2 H=E+T 1 3. d= a d+ [ (osinge +.acosq) =0, + maa ‘aperiodic motion, 170 area, 38 auiliary condition, 164 conservative system, 162, 168, 178 constraint, 163 damped oscillation, ~ differential equation, 100

Anda mungkin juga menyukai